Дб в разы. Перевод дБм в дБ (dBm в dB), взаимозависимость между мощностью и затуханием

06.03.2019

Области применения

Первоначально децибел использовался для измерения отношений энергетических (мощность , энергия) или силовых (напряжение, сила тока) величин. В принципе, с помощью децибелов можно измерять что угодно, но в настоящее время рекомендуется употреблять децибелы только для измерения уровня мощности и некоторых других связанных с мощностью величин. Так децибелы сегодня используются в акустике для измерения громкости звука и в электронике для измерения мощности электрического сигнала . Иногда в децибелах также измеряют динамический диапазон (например, звучания музыкальных инструментов). Также децибел является единицей звукового давления.

Измерение мощности

Как уже было сказано выше, изначально белы использовались для оценки отношения мощностей , поэтому в каноническом, привычном смысле величина, выраженная в белах, означает логарифмическое отношение двух мощностей и вычисляется по формуле:

величина в белах =

где P 1 / P 0 - отношение уровней двух мощностей, обычно измеряемой к т. н. опорной , базовой (взятой за нулевой уровень). Если говорить более точно, то это - «белы по мощности» . Тогда отношение двух величин в «децибелах по мощности» вычисляется по формуле:

величина в децибелах (по мощности) =

Измерение немощностных величин

Формулы для вычисления в децибелах разностей уровней немощностных (неэнергетических) величин, таких как напряжение или сила тока , отличаются от приведённой выше! Но в конечном итоге отношение этих величин, выраженное в децибелах, также выражается через отношение связанных с ними мощностей.

Так для линейной цепи справедливо равенство или

Отсюда видим, что а значит

откуда получаем равенство: которое представляет собой связь между «белами по мощности» и «белами по напряжению» в одной и той же цепи.

Из всего этого видим, что при сравнении величин напряжений (U 1 и U 2) или токов (I 1 и I 2) их отношения в децибелах выражаются формулами:

децибелы по напряжению = децибелы по току =

Можно подсчитать, что при измерении мощности изменению на 1 дБ соответствует приращение мощности (P 2 /P 1) в ≈1,25893 раза. Для напряжения или силы тока изменению на 1 дБ будет соответствовать приращение в ≈1,122 раза.

Пример вычислений

Предположим, что мощность P 2 в 2 раза больше начальной мощности P 1 , тогда

10 log 10 (P 2 /P 1) = 10 log 10 2 ≈ 3 дБ,

то есть изменение мощности на 3 дБ означает её увеличение в 2 раза. Аналогично изменение мощности в 10 раз:

10 log 10 (P 2 /P 1) = 10 log 10 10 = 10 дБ,

а в 1000 раз

10 log 10 (P 2 /P 1) = 10 log 10 1000 = 30 дБ,

И, наоборот, чтобы получить разы из децибел (dB), нужно

Для мощности - для напряжения (тока) .

Например, зная опорный уровень (P 1) и значение в дБ можно найти значение мощности, например, при P 1 = 1 мВт и известном отношении 20 дБ (dB):

Аналогично для напряжения, при U 1 = 2 В и отношении в 6 дБ:

Вычисления вполне реально производить в уме, для этого достаточно помнить примерную несложную таблицу (для мощностей):

1 дБ 1.25 3 дБ 2 6 дБ 4 9 дБ 8 10 дБ 10 20 дБ 100 30 дБ 1000

Сложению (вычитанию) значений дБ соответствует умножение (деление) самих отношений. Отрицательные значения дБ соответствуют обратным отношениям. Например, уменьшение мощности в 40 раз это 4*10 раз или −6 дБ-10 дБ= −16 дБ. Увеличение мощности в 128 раз это 2^7 или 3 дБ*7=21 дБ. Увеличение напряжения в 4 раза эквивалентно увеличению мощности в 4*4=16 раз, это 2^4 или 3 дБ*4=12 дБ.

Практическое применение

Поскольку децибел - не абсолютная, а относительная величина и вычисляется для различных физических величин по-разному (см. выше), то во избежание путаницы при использовании децибелов на практике существуют дополнительные договорённости.

чаще всего нужно знать отношение двух уровней (напряжений), выраженное в децибелах, есть несколько значений, которые легко запомнить:

6 дБ - отношение 2:1

20 дБ - отношение 10:1

40 дБ - отношение 100:1

60 дБ - отношение 1000:1

80 дБ - отношение 10000:1

100 дБ - отношение 100000:1

120 дБ - отношение 1000000:1

Промежуточные значения можно легко вычислить по формуле - 20*Lg(U1/U2), где U1 - уровень(напряжение) сигнала,U2 - уровень(напряжение) шума, напомним, что измерения проводятся средне-квадратичным милливольтметром, либо анализатором спектра с фильтром МЭК(А), где МЭК - Международная электротехническая комиссия

Зачем вообще применять децибелы и оперировать логарифмами, если то же самое можно выразить привычными процентами или долями? Представим себе, что в совершенно тёмной комнате включили лампочку некоторой светосилы. При этом, комната разительно отличается по виду до и после включения. Изменение освещённости, выраженное в дБ, тоже огромно, теоретически бесконечно. Допустим, что теперь включили ещё одну такую же лампочку. Теперь эффект будет совсем не тот, может быть даже человек не сразу заметит изменения, если её включить плавно. И в децибелах это будет всего 3 дБ. Итак, на практике, в децибелах удобно выполнять измерения как сильно меняющихся величин, так и почти постоянных.

Условные обозначения

Для различных физических величин одному и тому же числовому значению , выраженному в децибелах , могут соответствовать разные уровни сигналов (вернее разности уровней). Поэтому во избежание путаницы такие «конкретизированные» единицы измерения обозначают теми же буквами «дБ», но с добавлением индекса - общепринятого обозначения измеряемой физической величины. Например «дБВ» (децибел относительно вольта) или «дБмкВ» (децибел относительно микровольта), «дБВт» (децибел относительно ватта) и т. п. В соответствии с международным стандартом МЭК 27-3 при необходимости указать исходную величину ее значение помещают в скобках за обозначением логарифмической величины, например для уровня звукового давления: L P (re 20 µPA) = 20 dB; L P (исх. 20 мкПа) = 20 дБ

Применение в теории автоматического регулирования

Децибел также используется в теории автоматического регулирования и управления (ТАУ) и является одним из важнейших параметров при сравнении амплитуд выходного и входного сигналов.

Опорный уровень

Несмотря на то, что децибел служит для определения отношения двух величин, иногда децибелы используют и для измерения абсолютных значений. Для этого достаточно условиться, какой уровень измеряемой физической величины будет принят за опорный уровень (условный 0). На практике распространены следующие опорные уровни и специальные обозначения для них:

Во избежание путаницы желательно указывать опорный уровень явно, например −20 дБ (относительно 0,775 B) .

При пересчёте уровней мощностей в уровни напряжений и обратно надо обязательно учитывать сопротивление, являющиеся стандартным для данной задачи:

  • дБВ для 50-омной СВЧ -цепи соответствует (дБм−13 дБ);
  • дБмкВ для 50-омной СВЧ-цепи соответствует (дБм+107 дБ)
  • дБВ для 75-омной ТВ -цепи соответствует (дБм−11 дБ);
  • дБмкВ для 75-омной ТВ-цепи соответствует (дБм+109 дБ)

Следует чётко помнить математические правила:

  • перемножать или делить относительные единицы нельзя;
  • суммирование или вычитание относительных единиц производится независимо от их исходной размерности и соответствует умножению или делению абсолютных.

Например, подав на один конец 50-омного кабеля с коэффициентом передачи −6 дБ, мощность 0 дБм, что эквивалентно 1 мВт, или 0,22 В, или 107 дБмкВ, на выходе получим мощность −6 дБм, что эквивалентно 0,25 мВт (в 4 раза меньше по мощности) или 0,11 В (в два раза меньше по напряжению) или 101 дБмкВ (на те же 6 дБ меньше).

В децибелах удобно измерять коэффициенты затухания и усиления:

Зачем логарифмы? Так ведь и человеческое восприятие имеет логарифмический характер! Представь себе пакет с покупками массой 1 кг. Если к этой массе добавить ещё литр килограмм, то изменение массы будет очень даже ощутимо. Если этот же килограмм добавить к массе, скажем, 15 кг, то прирост массы будет заметен, но уже почти не будет ощущаться. А уж если этот килограмм добавить к целой тонне, то прирост будет и вовсе незаметен. Чтобы толкать автомобиль с литром сока и без оного, требуется приложить одинаковое усилие.

Кроме того, вспоминаем математику логарифмов, и видим, как упрощаются некоторые расчёты.

Мощность сигнала затухает в линии в 6,3 раза, на приёмной стороне усилитель повышает мощностью в 25 раз. Во сколько раз мощность сигнала на выходе усилителя будет больше или меньше, чем на выходе генератора?

Вроде не страшно, но чтобы вычислить, что сигнал на выходе будет почти в 4 раза сильнее, чем на входе тракта, потребуется калькулятор.

Складывать и вычитать намного проще! Опять же получаем результат, что мощность сигнала на выходе тракта будет почти вчетверо выше, чем на входе. Что из цифры +5,9 дБ следует, что мощность сигнала вчетверо выше, мы ещё убедимся немного ниже. А пока вспомним ещё одно соображение из математики логарифмов

  • Логарифм степени равен произведению показателя и логарифма основания

    Только что мы посчитали, во сколько раз мощность сигнала на выходе тракта отличается от подаваемой в тракт. Наверняка хочется знать величину этой мощности. Можно ли выразить сами величины в децибелах? Конечно можно! Для этого надо величину поделить на единицу.

    Теперь запомни несколько утверждений:

    • Изменение мощности в 2 раза — это 3 дБ
    • Изменение мощности в 3 раза — это 4.8 дБ
    • Изменение мощности в 10 раз — это 10 дБ
    • Изменение мощности в 100 раз — это 20 дБ

    Правильность этих утверждений легко проверить. И именно отсюда следует, что рост сигнала на 6 дБ (2 раза по 3 дБ) — это увеличение мощности в 4 раза (дважды 2 раза). А увеличиение мощности в 20 раз (10×2) — это увеличение на 13 дБ (10 + 3)

    . изменение мощности.

    Я намеренно писал выше только о мощностях. Мощность имеет квадратичную зависимость от напряжения и от тока, а изменение на 3 децибелла — это всегда и во всех случаях изменение мощности в 2 раза. Как мы помним, мощность зависит от квадрата напряжения или от квадрата тока:

    Бесплатная юридическая консультация:


    • Всё начинается с закона Ома:

    Помним, что логарифм степени есть произведение показателя степени и логарифма основания. Показатель степени — это двойка, и умножать надо не на 10, а на 20. Выразим 2 Вольта в децибел-вольтах, и 3 децибел-вольта в Вольтах:

    Просто и нестрашно!

    • В расчётах энергетических величин (мощность) фигурирует число 10
    • В расчётах силовых величин (напряжение, ток) фигурирует число 20

    Немного расчётов

    Порешаем немного расчётных задач, чтобы совсем уверенно ориентироваться в децибелах.

    1. Громкость звука

    Громкость звука тоже измеряется в децибелах. Помня о том, что децибел — это мера отношения двух величин, мы обязательно всегда уточняем, по отношению к чему измерены эти децибелы, т.е. где начало отсчёта. А в данном случае — по отношению к порогу слышимости человека: 2×10 -5 Н/м 2 . Ньютон — это системная единица силы, т.е. явно силовая величина, поэтому в расчётах фигурирует число 20. А давайте посчитаем, какую силу оказывает звуковое давление на барабанную перепонку в нашем ухе, при взлёте реактивного самолёта и при тихом разговоре.

    Бесплатная юридическая консультация:


    Что мы знаем:

    • Величины в децибелах выражены по отношению к 2×10 -5 Н/м 2
    • Площадь барабанной перепонки у человека около 55 мм 2 , или 5,5×10 -5 м 2
    • Табличная громкость реактивного самолётадБ на расстоянии 5 м
    • Табличная громкость тихого разговора — 50 дБ на расстоянии 1 м

    — Ура! Я нашел Ньютона!

    Ньютон хитро улыбнувшись отвечает:

    — Ошибся, умник! Это Ньютон на квадратный метр! ТЫ НАШЕЛ ПАСКАЛЯ.

    Посчитаем величину звукового давления в Паскалях, или Ньютонах на квадратный метр:

    Бесплатная юридическая консультация:


    • Пересчитаем наш реактивный самолёт

    Умножаем давление в Паскалях на площадь в квадратных метрах, и получим величину силы в Ньютонах:

    • Для реактивного самолёта

    Пересчитаем Ньютоны в более ощутимые грамм-силы:

    0,0011 Н × 102 гс/Н = 0,1122 гc

  • Звук негромкого разговора давит на барабанную перепонку с силоу

    0,Н × 102 гс/Н = 0,гс

  • Как говорится, почувствуйте разницу! И не забывайте, что механизм слуха более сложен, и звук мы воспринимаем не только барабанной перепонкой в глубине уха!

    2. Перевод уровня напряжения в мощность сигнала

    На работе мы часто измеряем уровни радиосигнала на антенном входе измерительного приёмника. А измерительный приёмник по своим метрологическим свойствам близок к селективному вольтметру, и измеренная величина исчисляется в децибел-микровольтах (дБмкВ). В то же время, часто в радиоизмерениях оперируют мощностью сигнала в точке приёма, нередко выраженной в децибел-милливаттах (дБм). Давайте пересчитаем одно в другое!

    • Ещё раз перепишем выражение, связывающее мощность и напряжение:

    И действительно, сигнал уровнем 70 дБмкВ (3.16 мВ) развивает на нагрузке 50 Ом мощность 0.2 мкВ, или -37 дБм

    И для пущего счастья, сделал онлайн-калькулятор, пересчитывающий напряжение в децибел-микровольтах в мощность в децибел-милливаттах и обратно (знаю-знаю, в интернете их и без меня бесчисленное множество! 🙂)

    Онлайн-калькулятор децибел

    Правила пользования просты до безобразия. Измени значение любой из величин, и все остальные значения будут пересчитаны автоматически.

    Decibel Meter - простой и быстрый измеритель уровня шума

    Decibel Meter — простое приложение для определения уровня шума в децибелах в его текущем, среднем и максимальном значении. Будет весьма полезным если захотите к примеру, найти район где уровень шума достаточно безопасен для нервов и слуха. Все полученные значения записываются в журнал, поэтому позже вы сможете их просматривать и сравнивать с текущей обстановкой.

    Приложение распространяется бесплатно, язык простого интерфейса английский, поддерживаются ОС Windows Phone 7.5 и 8.

    Бесплатная юридическая консультация:

    Онлайн-калькулятор децибел

    Что такое децибел (dB)? Это единица измерения отношения двух величин. Рассчитывается по формуле:

    где A dB - величина в децибелах, A - измеренная физическая величина, A 0 - величина, принятая за базис.

    В звукоинженерии в децибелах измеряется уровень звукового сигнала относительно номинального (0 dB), например -6 dB, что соответствует уровню почти вдвое меньше номинального. Перевести уровень звука в децибелах в проценты или «разы» очень легко при помощи этого онлайн-калькулятора:

    Бесплатная юридическая консультация:


    Уровень шума в помещении: 3 способа измерения

    Бытовая техника, автомобили на улице, соседи – из всего этого складывается суммарный шум в квартире. Как его измерить? И как изменится уровень шума, если в помещении установить бризер?

    Профессиональные шумомеры

    У этих инструментов много похожих названий: шумомеры, измерители шума, измерители уровня шума, измерители звука, измерители уровня звука.

    Тратиться на собственный шумомер нет смысла. Бюджетные приборы за рублей по точности не сильно отличаются от мобильных приложений и программ для измерения шума. А точные устройства стоят намного больше – дорублей.

    Если вам все же удалось достать профессиональный шумомер, то помните важный нюанс. На дисплее прибора отметка «0 дБА» не значит, что в вашей квартире абсолютная тишина. Ноль на экране говорит о том, что уровень шума в помещении настолько низкий или высокий, что не входит в рабочий диапазон именно этого прибора.

    Приложения и программы

    По сути, для измерения уровня шума нужен микрофон и специальная программа-анализатор. На ноутбуке, планшете или смартфоне есть микрофон. К стационарному компьютеру можно подключить внешний микрофон. Осталось скачать программу-анализатор.

    На компьютере можно измерить уровень шума бесплатной программой Decibel Reader. Также шумомеры есть во многих звукозаписывающих программах. Например, Audacity.

    Бесплатная юридическая консультация:


    На смартфоне или планшете микрофон, как правило, хуже, чем внешний микрофон, подключенный к компьютеру. Но и с ним можно провести достаточно точный замер уровня шума. Калибровка с профессиональными приборами показывает, что точность измерений на смартфоне может отличаться от профессионального оборудования всего на 5 децибел. Так что мобильными приложениями для оперативной работы пользуются даже специалисты по шумовой диагностике.

    Smart Tools – одно из приложений с функцией шумомера. Обратите внимание, что измерения не в дБА, а в дБ.

    «Обычный» децибел – это дБ, единица звукового давления. Но наше ухо по-разному воспринимает давление звуков разной частоты. Чтобы шумомер показывал реальный уровень шума, который слышит человек, в нем должен быть так называемый частотный фильтр А. С ним дБ превращается в тот самые дБА.

    В приложении Smart Tools нет частотного фильтра, но и без него можно получить общее представление об уровне шума.

    Другие приложения для измерения уровня шума:

    Бесплатная юридическая консультация:


    • MacOS: Decibel 10th, Decibel Meter Pro, dB Meter, Sound Level Meter
    • Android: Sound Meter, Decibel Meter, Noise Meter, deciBel
    • Windows: Decibel Meter Free, Cyberx Decibel Meter, Decibel Meter Pro

    в специализированных шумомерах (даже самых простых) параметры микрофона и обработчика сигнала согласованы. В смартфоне этого нет, поэтому точность измерения мобильным приложением всегда будет ниже, чем специальным прибором.

    Сравнительная оценка

    Известны примерные уровни шума холодильника, бризера, пылесоса, человеческой речи и других привычных источников шума. Единица измерения – акустические децибелы, дБА.

    Фактически этот способ – даже не измерение, а сравнительная оценка. Она дает грубое представление о том, насколько уровень шума у вас дома соответствует нормам.

    Для начала постарайтесь по максимуму снизить уровень шума в помещении. Плотно закройте окна и двери, чтобы избавиться от звуков с улицы, подъезда и других комнат. Выключите всё: телевизор, компьютер и другие «шумные» приборы.

    Возьмите обычную металлическую иголку и уроните ее на пол. Громкость звука, который вы услышали, примерно равна 15 дБА. Подойдите вплотную к работающему холодильнику или бризеру. Эти приборы работают с громкостью околодБА.

    Бесплатная юридическая консультация:


    Днем фоновый шум в квартире должен быть не громче 40 дБА, а на прилегающей территории не громче 70 дБА. Ночью (с 23 до 7 часов) для дома и улицы другие значения - 30 и 60 дБА соответственно. Это комфортная обстановка, и к ней надо стремиться.

    Громкость спокойной беседы - околодБА. Если из-за шума в квартире вам приходится повышать голос для общения, значит фоновый уровень шума вдвое выше нормы.

    Шпаргалка

    Напоследок - пользовательский шумовой тест бризера Tion O2. На видео показано, что с включенным бризером и закрытыми окнами уровень шума в квартире намного ниже, чем с открытыми окнами. Бризер, работающий на первой и второй скорости, фактически не меняет уровень фонового шума.

    Вначале включается компьютер, слышно, как он разгоняет винты, потом я включаю бризер, сначала на максимум, а затем снижаю до минимума. Затем выключаю бризер и открываю окно.

    Бесплатная юридическая консультация:


    Бесплатная подписка на полезные статьи

    Г. Москва, ул. Рабочая, 93, стр. 2

    Конвертер величин

    Уровень звука

    Конвертер значений в децибелах и отношения амплитуд и мощностей

    Отношение амплитуд и мощностей должно быть положительным числом.

    Как улучшить прием мобильного телефона?

    Подробнее об уровне звука

    Общие сведения

    Уровень звука определяет его громкость и используется в акустике - науке, изучающей уровень и другие свойства звука. Когда говорят о громкости, часто имеют в виду уменно уровень звука. Некоторые звуки очень неприятны и могут вызвать ряд психологических и физиологических проблем, в то время как другие звуки, например музыка, звук прибоя и пение птиц - действуют успокаивающее, нравятся людям и улучшают их настроение.

    Бесплатная юридическая консультация:


    Таблица значений в децибелах и отношений амплитуд и мощностей

    Эта таблица показывает как логарифмическая шкала позволяет описать очень большие и очень маленькие числа, представляющие отношения мощностей, энергий или амплитуд.

    Ухо человека обладает очень высокой чувствительностью и способно услышать звуки от шепота на расстоянии 10 метров до шума реактивных двигателей. Мощность звука петарды может быть в раз больше, чем самый слабый звук, который способно услышать человеческое ухо (20 микропаскалей). Это очень большая разница! Поскольку человеческое ухо способно различать такой большой диапазон громкостей звуков, для измерения силы звука используется логарифмическая шкала. На шкале в децибелах самый слабый звук, называемый порогом слышимости, имеет уровень 0 децибел. Звук, который громче порога слышимости в 10 раз, имеет уровень 20 децибел. Если звук в 30 раз громче порога слышимости, его уровень будет равен 30 децибелам. Ниже приведены примеры громкости различных звуков:

    • Порог слышимости - 0 дБ
    • Шепот - 20 дБ
    • Спокойный разговор на расстоянии 1 м - 50 дБ
    • Мощный пылесос на расстоянии 1 м - 80 дБ
    • Звук, при длительном воздействии которого возможно ухудшение слуха - 85 дБ
    • Портативный мультимедийный проигрыватель при полной громкости - 100 дБ
    • Болевой порог - 130 дБ
    • Турбореактивный двигатель истребителя на расстоянии 30 м - 150 дБ
    • Светозвуковая ручная граната M84 на расстоянии 1,5 м - 170 дБ

    Музыка

    Музыка, согласно археологам, украшает нашу жизнь на протяжении не менеелет. Она окружает нас везде - музыка присутствует во всех культурах, и, как считают ученые, объединяет нас с другими людьми - в обществе, в семье, в группе по интересам. Мамы поют малышам колыбельные; люди ходят на концерты; танцы, как народные, так и современные, проходят под музыку. Музыка привлекает нас своей закономерностью и ритмичностью, так как мы часто ищем порядок и четкость и в повседневной жизни.

    Шумовое загрязнение

    В отличие от музыки, некоторые звуки вызывают у нас очень неприятные ощущения. Шум, возникший из-за жизнедеятельности людей, который мешает людям или приносит вред животным, называется шумовым загрязнением. Он вызывает у людей и животных ряд психологических и физиологических проблем, таких как бессонница, усталость, нарушения кровяного давления, нарушение слуха при сильном шуме, и другие проблемы.

    Источники шума

    Шум может быть вызван множеством факторов. Транспорт - один из главных шумовых загрязнителей окружающей среды. Особенно много шума производят самолеты, поезда и автомобили. Оборудование на различных предприятиях в промышленной зоне также является источником шума. Люди, живущие возле ветряных турбин, часто жалуются на шум и связанные с ним недомогания. Ремонтные работы, особенно те, что связанны с использoванием отбойных молотков, обычно производят много шума. В некоторых странах люди держат собак, часто - в целях безопасности. Эти собаки, чаще всего те, что живут во дворе, лают, если рядом другие собаки и незнакомые люди. Это не так заметно днем, когда вокруг и так много шума, но очень хорошо слышно ночью. Шум в жилых районах также часто вызван громкой музыкой в домах, барах и ресторанах.

    Ветряные турбины

    По данным организаций, контролирующих работу компаний, добывающих электроэнергию с помощью ветряных турбин, низкочастотный шум, который они производят, мешает спать и вызывает головные боли и другие симптомы у людей, живущих рядом с турбинами. Эти проблемы настолько серьезны, что люди часто бросают свои дома и уезжают, чтобы избавиться от этого шума. Сторонники ветряной энергетики, наоборот, утверждают, что эти проблемы вызваны не шумом непосредственно, а эффектом ноцебо. То есть, проблемы вызваны не самим звуком а ожиданием того, что эти проблемы должны появиться. На данный момент не существует длительных исследований этого вопроса, позволяющих понять кто прав. Так как возможность шумового загрязнения - реальная угроза, то необходимо как можно скорее начать исследования влияния этого шума на людей. Даже если исследования покажут, что шум от турбин не влияет на жизнь людей, эти знания помогут жителям возле ветряных турбин избавиться от влияния эффекта ноцебо.

    Бесплатная юридическая консультация:


    Поезда

    Инженеры постоянно стараются усовершенствовать как сами поезда, так и железнодорожные пути, чтобы уменьшить шум, вызванный движением поездов. Большая часть шума образуется во время колебаний, образующихся при движении колес по рельсам. Кроме этого на поворотах колеса издают шум из-за проскальзывания колес относительно рельсов. Последнее неизбежно, но шум можно уменьшить. Эксперименты по уменьшению этого шума обычно проводятся на моделях колес и рельсов. Часто достаточно уменьшить вибрацию колеса и рельсов, что достигается при усовершенствовании их конструкции. Также, уменьшить шум помогают улучшенные конструкции тормозного механизма.

    Конструкция железной дороги в целом также влияет на шум. Например, установка противошумных барьеров, похожих на те, что ставят вокруг скоростных трасс, помогает уменьшить шум. Насыпи из гравия вокруг рельсов тоже поглощают звуки.

    Некоторое шумовое загрязнение, связанное с железными дорогами, неизбежно. Например, звуковая сигнальная система на железнодорожных переездах необходима, и помогает предотвратить аварии. В условиях плохой видимости именно благодаря ей пешеходы и водители знают о приближении поезда. Эта система также необходима для людей с плохим зрением.

    Самолеты

    Шум, вызванный самолетами, в основном образуется во время работы воздушно-реактивных и турбиновинтовых двигателей. Проблема шумового загрязнения существует как для пассажиров и экипажа, так и для тех, кто живет рядом с аэропортом. Шум в кабине самолета, когда его двигатели работают на полную мощность, достигает 80 децибелов. Чтобы немного уменьшить этот шум, некоторые пассажиры используют наушники с системой активного шумоподавления, описанные ниже.

    Законы во многих странах не требуют, чтобы самолеты летали не ниже определенной высоты, даже в жилых районах. Также мало где ограничивается общее время, которое самолет может находиться над определенным пространством. Обычно воздушное пространство открыто для самолетов 24 часа в сутки, независимо от того, жилая это зона или нет. При планировании аэропорта его часто стараются вынести за черту города, но это не всегда возможно, особенно в мегаполисах. Чтобы помощь в борьбе с шумом в некоторых странах для компаний, занимающимся авиаперевозками выпускаются, сборники рекомендаций по уменьшению шумового загрязнения.

    Автомобили

    Шумовое загрязнение, вызванное автомобилями - привычная проблема, особенно в городах. Обычно причины шума две. На больших скоростях он вызван движением шин по асфальту. Зимние шины летом, или езда на внедорожных автомобилях по скоростным трассам усиливают эту проблему. Это происходит потому, что зимние и внедорожные шины сконструированы так, чтобы обеспечить максимальную силу трения при движении, которая, в свою очередь, помогает сцеплению шины с дорожным покрытием, необходимому на обледенелой дороге или на бездорожье. По мере увеличения силы трения, соответственно увеличивается и шум.

    Бесплатная юридическая консультация:


    Если, наоборот, автомобили движутся медленно, то шум в основном вызван двигателем. Производители автомобилей постоянно стараются уменьшить этот шум. Он мешает не только пешеходам и окрестным жителям, но и самим водителям. Поэтому контролируют не только общий звук, издаваемый автомобилем, но и звук, проникающий в кабину - особенно в дорогих автомобилях. Для этого кабину звукоизолируют, а также используют систему активного шумоподавления. Для подавления шума используют звуковые волны, находящиеся в противофазе волнам, вызывающих шум. Этот метод активного шумоподавления используют и в других сферах, например для подавления шума в наушниках. Ниже он описан более подробно.

    На больших и скоростных трассах часто устанавливают звукоизоляционный барьер, который не дает шуму проезжающих машин распространяться за пределы трассы. Некоторые барьеры сконструированы так удачно, что человек, стоящий по другую его сторону от трассы, практически не слышит проезжающие машины. К сожалению, не все барьеры так хорошо сделаны. Некоторые блокируют звук только на уровне первого этажа, и совсем не защищают от шума людей, живущих в многоэтажных домах.

    Благодаря их конструкции, двигатели электромобилей намного тише двигателей автомобилей, работающих на бензине. Иногда электромобили передвигаются настолько тихо, что их не слышно пешеходам, поэтому для безопасности окружающих электромобили иногда снабжают устройством, которое производит шум вместо двигателя. Это необходимо для безопасности движения.

    Строительство и ремонтные работы

    Шум от строительства и ремонтных работ, например от ремонта трасс и железных дорог, часто способствует общему шумовому загрязнению. Ремонтные работы особенно часто проводят в то время, когда путями или дорогами пользуется наименьшее число людей, то есть, ночью. Один и тот же шум ночью мешает людям гораздо сильнее, не только потому, что его лучше слышно в тишине, но и потому, что в это время большинство людей спит. В большинстве случаев этот шум невозможно контролировать, и он неизбежен. Во многих странах компания, которая проводит строительные или ремонтные работы, должна вначале получить разрешение. В нем обычно указаны условия работы, например запрет на работы ночью, по выходным, или в праздники.

    Бытовой и прочий шум

    Шум в частных домах трудно регулировать с помощью законов, однако городские власти обычно регулируют шум в общественных местах. Так, например, в некоторых странах ограничивают или полностью запрещают частным лицам устраивать фейверки. В некоторых случаях фейверки разрешены только в определенные праздничные дни. Нарушителей обычно штрафуют. Городские власти также иногда ограничивают максимальный шум пиротехнических средств. В некоторых странах органы, которые следят за шумовым загрязнением в городе или районе, выпускают брошюры с советами жителям о том, как уменьшить количество бытового шума, который они производят. Например, в них советуют заранее сообщать соседям в случае предстоящих шумных мероприятий или работ. Советуют также делать ремонт и другие дела, которые производят много шума, в то время суток, когда большинство людей бодрствует, а также дрессировать собак, чтобы те меньше лаяли, и устанавливать шумную бытовую технику подальше от стен, смежных со стенами соседей. Если шум из соседних домов и квартир чрезмерно громок, то в ряде стран считается нормальным звонить в полицию с жалобами.

    Бесплатная юридическая консультация:


    Звукоизоляция в некоторых зданиях, особенно в многоквартирных домах, сделана плохо, поэтому покупая или снимая дом или квартиру необходимо хорошо проверить, насколько звук с улицы или из других квартир проникает внутрь. Для этого можно попробовать следующее:

    • Заранее попросите товарища выйти в коридор и сделать вид, что он кому-то звонит со своего сотового телефона. Таким образом можно узнать, насколько хорошо в квартире слышен шум из коридора.
    • Проверьте, не скрипит ли пол. Если скрипит, то скорее всего половицы плохо пригнаны друг к другу и будут скрипеть и в других местах, а также, вероятно - и этажом выше.
    • Постарайтесь пойти смотреть квартиру в самое шумное время суток. Так как это время в каждом микрорайоне разное, то стоит обойти улицы вокруг дома несколько раз в разное время, чтобы понять, когда на улице больше всего шума.
    • Если рядом школа, то вероятно - это будет утром и в то время, когда школьники возвращаются домой.
    • Если рядом большая трасса - то во время часа пик, или, наоборот, рано утром, когда в утренней тишине проезжают на большой скорости грузовики и машины. Осмотр района ночью поможет узнать, есть ли поблизости шумные заведения, например бары.

    Если, несмотря на тщательную проверку, вы обнаружили после переезда, что в квартире шумно, то попробуйте для уменьшения шума сделать следующее:

    • Ткани, ковры, гобелены, и другие звукопоглощающие материалы улучшают звукоизоляцию и тишину в комнатах. Ими стоит покрыть пол, стены, и если возможно - потолок. Также можно повесить шторы не только на окна, но и на стены - они не только уменьшат шум, но также послужат и украшением комнаты.
    • Шум легко передается по твердым предметам в результате их вибрации. Поэтому при покупке лучше выбирать мягкую мебель. Для уменьшения шума нужно также ограничить движение твердых предметов. Например, мебель можно накрыть тканью или скатертью.
    • Чтобы уменьшить вибрацию стен, к ним можно приставить тяжелые предметы, например книжные шкафы или серванты.

    В некоторых съемных квартирах хозяева требуют от жильцов, чтобы во всех комнатах на полу было ковровое покрытие. Если ваши соседи сверху сильно шумят и вы подозреваете, что у них нет ковров, то можно обратиться к домовладельцу, чтобы это проверить.

    Законодательство о шуме

    В некоторых странах шум регулируют соответствующими законами. Нарушения обычно грозят штрафами. В этом случае жители могут пожаловаться на шум в окрестностях в органы, ответственные за соблюдение порядка. Жалобу обычно рассматривают, и по возможности проверяют источник шума. В ряде стран в многоквартирных домах также часто существуют правила о шуме, например о том, можно ли и в какое время разрешено играть на музыкальных инструментах.

    Бесплатная юридическая консультация:


    Во многих городах, чтобы построить или открыть в жилом районе ресторан, бар, ночной клуб, или другое заведение, в которых играет громкая музыка, необходимо получить лицензию. В ней часто указывается, какой уровень звука допустим, и в какое время. В некоторых районах запрещают строить такие заведения, или разрешают, но с условием, что здание будет звукоизолировано. С шумовым загрязнением также помогает зонирование, то есть, деление города на зоны, такие как спальная, промышленная, и другие. В этом случае зоны с наибольшим шумовым загрязнением, например промышленные зоны с предприятиями и заводами, стараются разместить как можно дальше от жилых районов, больниц и школ.

    Измерение уровня звука

    Уровень звука измеряют, чтобы убедиться, что он не превышает нормы и соответствует требованиям выполняемой работы, например, что микрофоны обеспечивают достаточную громкость звука во время мероприятия. Такие измерения также необходимы для обеспечения безопасного уровня шума на рабочем месте.

    Шумомеры

    Если окружающий шум превышает 85 децибел, то высока вероятность повреждения слуха, особенно когда человек подвержен такому шуму в течение длительного времени. Болевой порог человека начинается с 115 децибел, но у некоторых людей он может быть и 140 децибел. То есть, даже если уровень звука грозит потерей слуха, люди этого не замечают. Именно поэтому в ситуациях, когда люди подвергаются воздействию громкого звука в течение длительного времени, уровень звука измеряют специальными приборами, чтобы убедиться, что этот уровень не превышает норму. Обычно это - шумомеры. Большинство из них портативны, и их можно приобрести по доступной цене.

    Звуковые дозиметры

    Если необходимо измерить не только уровень звука на данный момент, но и общую дозу шумового воздействия в течение определенного промежутка времени, используют звуковые дозиметры. Так как часто повреждение слуха происходит именно из-за длительного воздействия громких звуков, дозиметры помогают определить, нужно ли людям, работающим в условиях повышенного шума, носить защитные наушники или ушные пробки. Также удобно использовать дозиметры, если уровень звука в течении дня неодинаков. Обычно дозиметры прикрепляют к одежде самих работников, но не все приветствуют использование дозиметров на рабочем месте, так как с ними связано много проблем. Например, работники могут легко исказить данные, намеренно или случайно, особенно когда они видят индикатор уровня звука. Дозиметры также часто мешают работе, и даже могут зацепиться и попасть в оборудование. Это грозит не только сломанным оборудованием, но вероятно и несчастными случаями с работниками. По этой причине вместо дозиметров можно использовать шумомеры, измеряя уровень звука в разное время и в разных местах. С помощью этой информации создается шумовая карта, которая дает приблизительное представление о шумовом загрязнении на разных участках рабочего помещения. Это особенно полезно знать, если работники каждый день работают в одних и тех же местах. В последнее время производители дозиметров также стараются бороться с указанными выше проблемами, выпуская дозиметры меньшего размера, с короткими проводами или вообще без проводов, и часто без дисплея, чтобы работник не мог влиять на работу прибора, основываясь на текущей информации о шуме.

    Способы борьбы с шумом

    На заводах, в аэропортах и на других рабочих местах, где много шума, необходимо не только измерять, но и контролировать количество шума, который слышат работники, чтобы защитить их слух и предотвратить его потерю. Шум не только ухудшает слух, но и не дает людям сосредоточиться. Это мешает работе и подвергает их дополнительной опасности, так как по невнимательности они могут не услышать аварийную сигнализацию из-за шума, что может привести к несчастному случаю. К тому же, в шумном помещении неприятно находиться и работать, поэтому звук контролируют еще и для комфорта работников. Не всегда есть возможность воспользоваться шумомером. В такой ситуации действует простое правило: если для того, чтобы быть услышанным, приходится кричать - то это значит, что помещение слишком шумное, и этот шум необходимо уменьшать.

    Бесплатная юридическая консультация:


    Есть два основных способа борьбы с шумом: шумоизоляция или шумоподавление с помощью противодействующего шума. Первый метод - пассивный, а второй - активный. Какой из двух методов использовать - решают в зависимости от ситуации, а иногда используют оба сразу. Также можно одновременно использовать сразу несколько способов пассивного шумоподавления или блокирования шума. Например, команды наземного технического обслуживания в аэропортах часто используют ушные пробки и наушники с пассивным шумоподавлением одновременно.

    Иногда на заводах и фабриках также используются звукопоглотители. Они предотвращают усиление звука в помещении и его отражение от стен и других поверхностей. Для этого звукопоглотители изготавливают из материалов, хорошо поглощающих звук.

    Пассивное шумоподавление

    Для пассивного шумоподавления используют материалы, которые хорошо поглощают звук. Большинство приведенных выше советов об уменьшении шума в квартире основаны именно на этом принципе. Звукопоглащающие материалы, используемые в наушниках - это вспененные полимеры.

    Активное шумоподавление

    С помощью активного шумоподавления можно уменьшить окружающий шум примерно на 20 децибел. Принцип активного подавления звука заключается в том, что входящая звуковая волна гасится при помощи исходящей волны с одинаковой амплитудой, но с противоположной фазой. Исходящий шум создают наушники.

    То, что происходит в этом случае со звуком, можно продемонстрировать с помощью примера о качелях. Когда один человек толкает качели вперед, а другой, с той же амплитудой начнет качать их назад, то эти толчки будут в противофазе. Когда две волны находятся в противофазе, то их общая сумма равна нулю. То есть, в случае с качелями - они перестанут качаться.

    Бесплатная юридическая консультация:


    Чтобы правильно блокировать звук, шумоподавляющие устройства сначала должны определить амплитуду и частоту входящих звуковых волн, чтобы потом создать аналогичные волны в противофазе. Такие устройства хорошо работают с монотонным повторяющимся звуком, который легко предсказать. Если же звук спонтанный и все время меняется, то шумоподавляющие устройства неэффективны. Входящий звук принимается в таких устройствах, например наушниках, на встроенный микрофон. Кроме кабин последних моделей автомобилей и бытовых наушников, активное шумоподавление используется в некоторых защитных наушниках для работников аэропортов.

    Поддержка защитных средств в рабочем состоянии

    Несмотря на то, что работодатели во многих странах обязаны предоставить своим работникам персональное средства защиты слуха, например наушники и ушные пробки, всегда лучше проверять их перед использованием, чтобы убедиться, что они в рабочем состоянии и нигде нет трещин. Это особенно важно потому, что иногда происходят ошибки, и неисправное снаряжение может быть не замечено при его проверке.

    Unit Converter articles were edited and illustrated by Анатолий Золотков

    Вас могут заинтересовать и другие конвертеры из группы «Акустика - звук»:

    Вы затрудняетесь в переводе единицы измерения с одного языка на другой? Коллеги готовы вам помочь. Опубликуйте вопрос в TCTerms и в течение нескольких минут вы получите ответ.

    Акустика - звук

    Уровень звука

    Звук представляет собой упругие волны, распространяющиеся в упругой среде (твердое тело, жидкость или газ) и создающие в ней механические колебания. Звуковые волны являются одним из примеров множества колебательных процессов.

    Децибел (дБ) - единица измерения уровня звука, уровней мощности или амплитуды электрических сигналов путем сравнения их с заданным уровнем с применением к полученному отношению логарифмического масштаба. Более широко децибел можно определить как логарифмическую безразмерную единицу отношения уровней к некоторому опорному уровню, а также затуханий и усилений. Величина, выраженная в децибелах, численно равна десятичному логарифму безразмерного отношения физической величины к одноимённой физической величине, принимаемой за исходную, умноженному на десять. Децибел равен одной десятой бела, который используется редко. Изменение мощности в 100 раз представляется как изменение в 20 дБ. Изменение на 3 дБ приблизительно соответствует изменению мощности в два раза. В науке и технике, в частности, в электронике и радиотехнике, децибел применяется для измерения отношения некоторых величин - «энергетических» (мощности, энергии, плотности потока мощности) или «амплитудных» (силы тока, напряжения, силы звука).

    В акустике децибел обычно используется для указания громкости звука относительно уровня 0 децибел, который определяется как уровень звукового давления 20 микропаскалей. Обычно это отношение указывается для мощности.

    Непер (Нп)- логарифмическая безразмерная единица измерения отношения двух уровней, затуханий или усилений. Непер не входит в систему единиц СИ. Разница между белом и непером заключается в том, что отношение величин, выраженное в белах или децибелах, предполагает использование десятичных логарифмов, в то время как для отношения в неперах используются натуральные логарифмы (по основанию е).

    Использование конвертера «Уровень звука»

    На этих страницах размещены конвертеры единиц измерения, позволяющие быстро и точно перевести значения из одних единиц в другие, а также из одной системы единиц в другую. Конвертеры пригодятся инженерам, переводчикам и всем, кто работает с разными единицами измерения.

    Пользуйтесь конвертером для преобразования нескольких сотен единиц в 76 категориях или несколько тысяч пар единиц, включая метрические, британские и американские единицы. Вы сможете перевести единицы измерения длины, площади, объема, ускорения, силы, массы, потока, плотности, удельного объема, мощности, давления, напряжения, температуры, времени, момента, скорости, вязкости, электромагнитные и другие.

    Примечание. В связи с ограниченной точностью преобразования возможны ошибки округления. В этом конвертере целые числа считаются точными до 15 знаков, а максимальное количество цифр после десятичной запятой или точки равно 10.

    Для представления очень больших и очень малых чисел в этом калькуляторе используется компьютерная экспоненциальная запись, являющаяся альтернативной формой нормализованной экспоненциальной (научной) записи, в которой числа записываются в форме a · 10 x . Например: = 1,103 · 10 6 = 1,103E+6. Здесь E (сокращение от exponent ) - означает «· 10^», то есть «. умножить на десять в степени. » . Компьютерная экспоненциальная запись широко используется в научных, математических и инженерных расчетах.

    • Выберите единицу, с которой выполняется преобразование, из левого списка единиц измерения.
    • Выберите единицу, в которую выполняется преобразование, из правого списка единиц измерения.
    • Введите число (например, «15») в поле «Исходная величина».
    • Результат сразу появится в поле «Результат» и в поле «Преобразованная величина».
    • Можно также ввести число в правое поле «Преобразованная величина» и считать результат преобразования в полях «Исходная величина» и «Результат».

    Мы работаем над обеспечением точности конвертеров и калькуляторов TranslatorsCafe.com, однако мы не можем гарантировать, что они не содержат ошибок и неточностей. Вся информация предоставляется «как есть», без каких-либо гарантий. Условия.

    Если вы заметили неточность в расчётах или ошибку в тексте, или вам необходим другой конвертер для перевода из одной единицы измерения в другую, которого нет на нашем сайте - напишите нам!

    ANVICA Software Development 2002-2018.

    ЧТО ТАКОЕ ДЕЦИБЕЛЫ?

    Универсальные логарифмические единицы децибелы широко используются при количественных оценках параметров различных аудио и видео устройств в нашей стране и за рубежом. В радиоэлектронике, в частности, в проводной связи, технике записи и воспроизведения информации децибелы являются универсальной мерой.

    Децибел - не физическая величина, а математическое понятие

    В электроакустике децибел служит по существу единственной единицей для характеристики различных уровней - интенсивности звука, звукового давления, громкости, а также для оценки эффективности средств борьбы с шумами.

    Децибел - специфическая единица измерений, не схожая ни с одной из тех, с которыми приходится встречаться в повседневной практике. Децибел не является официальной единицей в системе единиц СИ, хотя, по решению Генеральной конференции по мерам и весам, допускается его применение без ограничений совместно с СИ, а Международная палата мер и весов рекомендовала включить его в эту систему.

    Децибел - не физическая величина, а математическое понятие.

    В этом отношении у децибел есть некоторое сходство с процентами. Как и проценты, децибелы безразмерны и служат для сравнения двух одноименных величин, в принципе самых различных, независимо от их природы. Следует отметить, что термин «децибел» всегда связывают только с энергетическими величинами, чаще всего с мощностью и, с некоторыми оговорками, с напряжением и током.

    Децибел (русское обозначение - дБ, международное - dB) составляет десятую часть более крупной единицы - бела 1 .

    Бел - это десятичный логарифм отношения двух мощностей. Если известны две мощности Р 1 и Р 2 , то их отношение, выраженное в белах, определяется формулой:

    Физическая природа сравниваемых мощностей может быть любой - электрической, электромагнитной, акустической, механической, - важно лишь, чтобы обе величины были выражены в одинаковых единицах - ваттах, милливаттах и т. п.

    Напомним вкратце, что такое логарифм. Любое положительное 2 число, как целое, так и дробное, можно представить другим числом в определенной степени.

    Так, например, если 10 2 = 100, то 10 называют основанием логарифма, а число 2 - логарифмом числа 100 и обозначают log 10 100=2 или lg 100 = 2 (читается так: «логарифм ста при основании десять равен двум»).

    Логарифмы с основанием 10 называются десятичными логарифмами и применяются чаще всего. Для чисел, кратных 10, этот логарифм численно равен количеству нулей за единицей, а для остальных чисел вычисляется на калькуляторе или находится по таблицам логарифмов.

    Логарифмы с основанием е = 2,718... называются натуральными. В вычислительной технике обычно применяются логарифмы с основанием 2.

    Основные свойства логарифмов:

    Разумеется, эти свойства справедливы и для десятичных и натуральных логарифмов. Логарифмический способ представления чисел часто оказывается очень удобным, так как позволяет подменять умножение - сложением, деление - вычитанием, возведение в степень умножением, а извлечение корня - делением.

    На практике бел оказался слишком крупной величиной, например, любые отношения мощностей в границах от 100 до 1000 укладываются в пределах одного бела - от 2 Б до 3 Б. Поэтому для большей наглядности решили число, показывающее количество бел, умножать на 10 и полученное произведение считать показателем в децибелах, т. е., например, 2 Б = 20 дБ, 4,62 Б = 46,2 дБ и т. д.

    Обычно отношение мощностей выражают сразу в децибелах по формуле:

    Действия с децибелами не отличаются от операций с логарифмами.

    2 дБ = 1 дБ + 1 дБ → 1,259 * 1,259 = 1,585;
    3 дБ → 1,259 3 = 1,995;
    4 дБ → 2,512;
    5 дБ → 3,161;
    6 дБ → 3,981;
    7 дБ → 5,012;
    8 дБ → 6,310;
    9 дБ → 7,943;
    10 дБ → 10,00.

    Знак → означает «соответствует».

    Подобным образом можно составить таблицу и для отрицательных значений децибел. Минус 1 дБ характеризует убывание мощности в 1/0,794 = 1,259 раза, т. е. тоже примерно на 26%.

    Запомните, что:

    ⇒ Если Р 2 1 т. е. P 2 /P 1 =1 , то N дБ = 0 , так как lg 1=0 .

    ⇒ Если P 2 > P l , то число децибел положительно.

    ⇒ Если Р 2 < P 1 , то децибелы выражаются отрицательными числами.

    Положительные децибелы часто называют децибелами усиления. Отрицательные децибелы, как правило, характеризуют потери энергии (в фильтрах, делителях, длинных линиях) и называются децибелами затухания или потерь.

    Между децибелами усиления и затухания существует простая зависимость: одинаковому числу децибел с разными знаками соответствуют обратные числа отношений. Если, например, отношению Р 2 1 = 2 → 3 дБ , то –3 дБ → 1/2 , т. е. 1 / Р 2 1 = Р 1 2

    ⇒ Если Р 2 1 представляет степень десяти, т. е. Р 2 1 = 10 k , где k - любое целое число (положительное или отрицательное), то NдБ = 10k , так как lg 10 k = k .

    ⇒ Если Р 2 или Р 1 равно нулю, то выражение для NдБ теряет смысл.

    И еще одна особенность: кривая, определяющая значения децибел в зависимости от отношений мощностей, вначале быстро растет, затем ее рост замедляется.

    Зная число децибел, соответствующих одному отношению мощностей, можно произвести пересчет для другого - близкого или кратного отношения. В частности, для отношений мощностей, различающихся в 10 раз, число децибел отличается на 10 дБ. Эту особенность децибел следует хорошо понять и твердо запомнить - она является одной из основ всей системы

    К достоинствам системы децибел относят:

    ⇒ универсальность, т. е. возможность использования при оценке различных параметров и явлений;

    ⇒ огромные перепады преобразуемых чисел - от единиц и до миллионов - отображаются в децибелах числами первой сотни;

    ⇒ натуральные числа, представляющие степени десяти, выражаются в децибелах числами, кратными десяти;

    ⇒ взаимообратные числа выражаются в децибелах равными числами, но с разными знаками;

    ⇒ в децибелах могут быть выражены как отвлеченные, так и именованные числа.

    К недостаткам системы децибел относят:

    ⇒ малую наглядность: для преобразования децибел в отношения двух чисел или выполнения обратных действий требуется проведение расчетов;

    ⇒ отношения мощностей и отношения напряжений (или токов) пересчитываются в децибелы по разным формулам, что иногда ведет к ошибкам и путанице;

    ⇒ децибелы могут отсчитываться только относительно не равного нулю уровня; абсолютный нуль, например 0 Вт, 0 В, децибелами не выражается.

    Зная число децибел, соответствующих одному отношению мощностей, можно произвести пересчет для другого - близкого или кратного отношения. В частности, для отношений мощностей, различающихся в 10 раз, число децибел отличается на 10 дБ. Эту особенность децибел следует хорошо понять и твердо запомнить - она является одной из основ всей системы.

    Сравнение двух сигналов путем сопоставления их мощностей не всегда бывает удобным, так как для непосредственного измерения электрической мощности в диапазоне звуковых и радиочастот требуются дорогие и сложные приборы. На практике при работе с аппаратурой гораздо проще измерять не мощность, которая выделяется на нагрузке, а падение напряжения на ней, а в некоторых случаях - протекающий ток.

    Зная напряжение или ток и сопротивление нагрузки, легко определить мощность. Если измерения проводятся на одном и том же резисторе, то:

    Этими формулами очень часто пользуются практике, но обратите внимание, что если напряжения или токи измеряются на разных нагрузках, эти формулы не работают и следует использовать другие, более сложные зависимости.

    Пользуясь приемом, который был использован при составлении таблицы децибел мощности, можно аналогично определить, чему равен 1 дБ отношения напряжений и токов. Положительный децибел будет равен 1,122, а отрицательный децибел будет равен 0,8913, т.е. 1 дБ напряжения или тока характеризует возрастание или убывание этого параметра примерно на 12% по отношению к первоначальному значению.

    Формулы выводились в предположении, что сопротивления нагрузок имеют активный характер и между напряжениями или токами нет фазового сдвига. Строго говоря, следовало бы рассматривать общий случай и учитывать для напряжений (токов) наличие угла сдвига по фазе, а для нагрузок не только активное, но полное сопротивление, включая и реактивные составляющие, однако это существенно только на высоких частотах.

    Полезно запомнить некоторые часто встречающиеся на практике значения децибел и характеризующие их отношения мощностей и напряжений (токов), приведенные в табл. 1.

    Таблица 1. Часто встречающиеся значения децибел мощности и напряжения

    Пользуясь этой таблицей и свойствами логарифмов легко подсчитать, чему соответствуют произвольные значения логарифм. Например, 36 дБ мощности можно представить как 30+3+3, что соответствует 1000*2*2 = 4000. Тот же самый результат мы получим, представив 36 как 10+10+10+3+3 → 10*10*10*2*2 = 4000.

    СОПОСТАВЛЕНИЕ ДЕЦИБЕЛ С ПРОЦЕНТАМИ

    Ранее отмечалось, что понятие децибел имеет некоторое сходство с процентами. Действительно, так как в процентах выражается отношение какого-то числа к другому, условно принятому за сто процентов, отношение этих чисел также можно представить в децибелах при условии, что оба числа характеризуют мощность, напряжение или ток. Для отношения мощностей:

    Для отношения напряжений или токов:

    Можно также вывести формулы для пересчета децибел в проценты отношения:

    В табл. 2 дан перевод некоторых, наиболее часто встречающихся значений децибел в проценты отношений. Различные промежуточные значения можно найти по номограмме на рис. 1.


    Рис. 1. Перевод децибел в проценты отношений по номограмме

    Таблица 2. Перевод децибел в проценты отношений

    Рассмотрим два практических примера, поясняющих перевод процентного отношения в децибелы.

    Пример 1. Какому уровню гармоник в децибелах по отношению к уровню сигнала основной частоты соответствует коэффициент нелинейных искажений в 3%?

    Воспользуемся рис. 1. Через точку пересечения вертикальной линии 3% с графиком «напряжение» проведем горизонтальную линию до пересечения с вертикальной осью и получим ответ: –31 дБ.

    Пример 2. Какому ослаблению напряжения в процентах соответствует его изменение на –6 дБ?

    Ответ. На 50% первоначальной величины.

    В практических расчетах дробную часть численного значения децибел часто округляют до целого числа, однако при этом в результаты расчетов вносится дополнительная погрешность.

    ДЕЦИБЕЛЫ В РАДИОЭЛЕКТРОНИКЕ

    Рассмотрим несколько примеров, поясняющих методику использования децибел в радиоэлектронике.

    Затухание в кабеле

    Потери энергии в линиях и кабелях на единицу длины характеризуются коэффициентом затухания α, который при равном входном и выходном сопротивлениях линии определяется в децибелах:

    где U 1 - напряжение в произвольном сечении линии; U 2 - напряжение в другом сечении, отстоящем от первого на единицу длины: 1 м, 1 км и т. д. Например, высокочастотный кабель типа РК-75-4-14 имеет на частоте 100 МГц коэффициент затухания α, = –0,13 дБ/м, кабель витой пары категории 5 на той же частоте имеет затухание порядка –0,2 дБ/м, а у кабеля категории 6 несколько меньше. График затухания сигнала в неэкранированном кабеле витой пары показан на рис. 2.


    Рис. 2. График затухания сигнала в неэкранированном кабеле витой пары

    Оптоволоконные кабели имеют существенно более низкие величины затухания в диапазоне от 0,2 до 3 дБ при длине кабеля в 1000 м. Все оптические волокна имеют сложную зависимость затухания от длины волны, которая имеет три «окна прозрачности» 850 нм, 1300 нм и 1550 нм. «Окно прозрачности» означает наименьшие потери при максимальной дальности передачи сигнала. График затухания сигнала в оптоволоконных кабелях показан на рис. 3.


    Рис. 3. График затухания сигнала в оптоволоконных кабелях

    Пример 3. Найти, каким будет напряжение на выходе отрезка кабеля РК-75-4-14 длиной l = 50 м, если ко входу его приложено напряжение 8 В частоты 100 МГц. Сопротивление нагрузки и волновое сопротивление кабеля равны, или, как говорят, согласованы между собой.

    Очевидно, что затухание, вносимое отрезком кабеля, составляет K = –0,13 дБ/м * 50 м = –6,5 дБ. Это значение децибел примерно соответствует отношению напряжений 0,47. Значит, напряжение на выходном конце кабеля U 2 = 8 В * 0,47 = 3,76 В.

    Этот пример иллюстрирует очень важное положение: потери в линии или кабеле с ростом их длины возрастают чрезвычайно быстро. Для отрезка кабеля длиной в 1 км затухание составит уже –130 дБ, т. е. сигнал будет ослаблен более чем в триста тысяч раз!

    Затухание в значительной мере зависит от частоты сигналов - в диапазоне звуковых частот оно будет гораздо меньше, чем в видео диапазоне, но логарифмический закон затухания будет тот же, и при большой длине линии ослабление будет существенным.

    Усилители звуковой частоты

    В усилители звуковой частоты с целью повышения их качественных показателей обычно вводится отрицательная обратная связь. Если коэффициент усиления устройства по напряжению без обратной связи равен К , а с обратной связью К ОС то число, показывающее, во сколько раз изменяется коэффициент усиления под действием обратной связи, называют глубиной обратной связи . Ее обычно выражают в децибелах. В работающем усилителе коэффициенты К и К ОС определяются экспериментально, если только усилитель не возбуждается при разомкнутой петле обратной связи. При проектировании усилителя сначала вычисляют К , а затем определяют значение К ОС следующим образом:

    где β - коэффициент передачи цепи обратной связи, т. е. отношение напряжения на выходе цепи обратной связи к напряжению на ее входе.

    Глубина обратной связи в децибелах может быть рассчитана по формуле:

    Стереофонические устройства по сравнению с монофоническими должны удовлетворять дополнительным требованиям. Эффект объемного звучания обеспечивается только при хорошем разделении каналов, т. е. при отсутствии проникновения сигналов из одного канала в другой. В практических условиях это требование полностью удовлетворить не удается, и взаимное просачивание сигналов имеет место, главным образом, через узлы, общие для обоих каналов. Качество разделения по каналам характеризуется так называемым переходным затуханием а ПЗ Мерой переходного затухания в децибелах служит отношение выходных мощностей обоих каналов, когда входной сигнал подается только на один канал:

    где Р Д - максимальная выходная мощность действующего канала; Р СВ - выходная мощность свободного канала.

    Хорошему разделению каналов соответствует переходное затухание 60-70 дБ, отличному –90-100 дБ.

    Шум и фон

    На выходе любого приемно-усилительного устройства даже при отсутствии полезного входного сигнала можно обнаружить переменное напряжение, которое вызвано собственными шумами устройства. Причины, вызывающие собственные шумы, могут быть как внешними - за счет наводок, плохой фильтрации напряжения питания, так и внутренними, обусловленными собственными шумами радиокомпонентов. Сильнее всего сказываются шумы и, помехи, возникающие во входных цепях и в первом усилительном каскаде, так как они усиливаются всеми последующими каскадами. Собственные шумы ухудшают реальную чувствительность приемника или усилителя.

    Количественная оценка шумов осуществляется несколькими способами.

    Простейший состоит в том, что все шумы, независимо от причины и места их возникновения, пересчитываются ко входу, т. е. напряжение шумов на выходе (при отсутствии входного сигнала) делится на коэффициент усиления:

    Это напряжение, выраженное в микровольтах, и служит мерой собственных шумов. Однако для оценки устройства с точки зрения помех важно не абсолютное значение шумов, а отношение между полезным сигналом и этим шумом (отношение сигнал/шум), так как полезный сигнал должен надежно выделяться на фоне помех. Отношение сигнал/шум обычно выражают в децибелах:

    где Р с - заданная или номинальная выходная мощность полезного сигнала вместе с шумом; Р ш - выходная мощность шумов при выключенном источнике полезного сигнала; U c - напряжение сигнала и шумов на нагрузочном резисторе; U Ш - напряжение шумов на том же резисторе. Так получается т.н. «невзвешенное» («unweighted») отношение сигнал/шум.

    Часто в параметрах аудиоаппаратуры приводится отношение сигнал/шум, измеренное со взвешивающим фильтром («weighted»). Фильтр позволяет учесть разную чувствительность слуха человека к шуму на разных частотах. Чаще всего используется фильтр типа А, в этом случае в обозначении обычно указывается единица измерения «дБА» («dBA»). Использование фильтра дает обычно лучшие количественные результаты, чем для невзвешенного шума (обычно отношение сигнал/шум получается на 6-9 дБ больше), поэтому (из маркетинговых соображений) производители аппаратуры чаще указывают именно «взвешенное» значение. Подробнее о взвешивающих фильтрах см. ниже в разделе «Шумомеры».

    Очевидно, что для успешной эксплуатации устройства отношение сигнал/шум должно быть выше какого-то минимально допустимого значения, которое зависит от назначения и требований, предъявляемых к устройству. Для аппаратуры класса Hi-Fi этот параметр должен быть не менее 75 дБ, для аппаратуры Hi-End - не менее 90 дБ.

    Иногда на практике пользуются обратным отношением, характеризуя им уровень шумов относительно полезного сигнала. Уровень шумов выражается тем же числом децибел, что и отношение сигнал/шум, но с отрицательным знаком.

    В описаниях приемно-усилительной аппаратуры иногда фигурирует термин уровень фона, который характеризует в децибелах отношение составляющих напряжения фона к напряжению, соответствующему заданной номинальной мощности. Составляющие фона кратны частоте питающей сети (50, 100, 150 и 200 Гц) и при измерении выделяются из общего напряжения помех при помощи полосовых фильтров.

    Отношение сигнал/шум не позволяет, однако, судить о том, какая часть шумов обусловлена непосредственно элементами схемы, а какая внесена в результате несовершенства конструкции (наводки, фон). Для оценки шумовых свойств радиокомпонентов вводится понятие коэффициента (фактора) шума . Коэффициент шума оценивается по мощности и также выражается в децибелах. Характеризовать этот параметр можно следующим образом. Если на входе устройства (приемника, усилителя) одновременно действуют полезный сигнал мощностью Р с и шумы мощностью Р ш , то отношение сигнал/шум на входе будет с ш )вх После усиления отношение с ш )вых окажется меньше, так как к входным шумам добавятся и усиленные собственные шумы усилительных каскадов.

    Коэффициентом шума называют выраженное в децибелах отношение:

    где К р - коэффициент усиления по мощности.

    Следовательно, коэффициент шума представляет отношение мощности шумов на выходе к усиленной мощности шумов, действующих на входе.

    Значение Рш.вх определяется расчетным путем; Рш.вых измеряется, а К р обычно. известно из расчета или после измерения. Идеальный с точки зрения шумов усилитель должен усиливать только полезные сигналы и не должен вносить дополнительные шумы. Как следует из уравнения, для подобного усилителя коэффициент шума F Ш = 0 дБ .

    Для транзисторов и ИС, предназначенных для работы в первых каскадах усилительных устройств, коэффициент шума регламентируется и приводится в справочниках.

    Напряжение собственных шумов определяет и другой важный параметр многих усилительных устройств - динамический диапазон.

    Динамический диапазон и регулировки

    Динамическим диапазоном называется выраженное в децибелах отношение максимальной неискаженной выходной мощности к ее минимальному значению, при котором, еще обеспечивается допустимое отношение сигнал/шум:

    Чем меньше уровень собственных шумов и чем выше неискаженная выходная мощность, тем шире динамический диапазон.

    Аналогичным образом определяется и динамический диапазон источников звука - оркестра, голоса, только здесь минимальная мощность звука определяется шумовым фоном. Чтобы устройство могло передать без искажений как минимальную, так и максимальную амплитуды входного сигнала, его динамический диапазон должен быть не меньше динамического диапазона сигнала. В случаях, когда динамический диапазон входного сигнала превышает динамический диапазон устройства, его искусственно сжимают. Так поступают, например, при звукозаписи.

    Эффективность действия ручного регулятора громкости проверяется при двух крайних положениях регулятора. Сначала при регуляторе в положении максимальной громкости на вход усилителя звуковой частоты подается напряжение частотой 1 кГц такой величины, чтобы на выходе усилителя установилось напряжение, соответствующее некоторой заданной мощности. Затем ручку регулятора громкости переводят на минимальную громкость, а напряжение на входе усилителя поднимают до тех пор, пока напряжение на выходе снова не станет равным первоначальному. Отношение входного напряжения при регуляторе в положении минимальной громкости к входному напряжению при максимальной громкости, выраженное в децибелах, является показателем работы регулятора громкости.

    Приведенными примерами далеко не исчерпываются практические случаи приложения децибел к оценке параметров радиоэлектронных устройств. Зная общие правила, применения этих единиц, можно понять, как они используются в других, не рассмотренных здесь условиях. Встретившись с незнакомым термином, определенным в децибелах, следует отчетливо представить, отношению каких двух величин он соответствует. В одних случаях это понятно из самого определения, в других случаях связь между составляющими сложнее, и, когда нет четкой ясности, следует обратиться к описанию методики измерения во избежание серьезных ошибок.

    Оперируя с децибелами, следует всегда обращать внимание на то, отношению каких единиц - мощности или напряжения - соответствует каждый конкретный случай, т. е. какой коэффициент - 10 или 20 - должен стоять перед знаком логарифма.

    ЛОГАРИФМИЧЕСКИЙ МАСШТАБ

    Логарифмическая система, в том числе и децибелы, часто применяется при построении амплитудно-частотных характеристик (АЧХ) - кривых, изображающих зависимость коэффициента передачи различных устройств (усилителей, делителей, фильтров) от частоты внешнего воздействия. Для построения частотной характеристики расчетным или опытным путем определяется ряд точек, характеризующих выходное напряжение или мощность при неизменном входном напряжении на разных частотах. Плавная кривая, соединяющая эти точки, характеризует частотные свойства устройства или системы.

    Если по оси частот численные значения откладывать в линейном масштабе, т. е. пропорционально их фактическим значениям, то такая частотная характеристика окажется неудобной для пользования и не будет наглядной: в области низших частот она сжата, а высших - растянута.

    Частотные характеристики строятся обычно в так называемом логарифмическом масштабе. По оси частот в удобном для работы масштабе откладываются величины, пропорциональные не самой частоте f , а логарифму lgf/f o , где f о - частота, соответствующая началу отсчета. Против отметок на оси надписываются значения f . Для построения логарифмических АЧХ используют специальную логарифмическую миллиметровую бумагу.

    При проведении теоретических расчетов обычно пользуются не просто частотой f , а величиной ω = 2πf которую называют круговой частотой.

    Частота f о , соответствующая началу отсчета, может быть сколь угодно малой, но не может быть равной нулю.

    По вертикальной оси откладываются в децибелах либо в относительных числах отношения коэффициентов передачи при различных частотах к его максимальному либо среднему значению.

    Логарифмический масштаб позволяет на небольшом отрезке оси отобразить широкий диапазон частот. На такой оси одинаковым отношениям двух частот соответствуют равные по длине участки. Интервал, характеризующий рост частоты в десять раз, называют декадой ; двукратному отношению частот соответствует октава (этот термин заимствован из теории музыки).

    Частотный диапазон с граничными частотами f H и f В занимает в декадах полосу f B /f H = 10m , где m - число декад, а в октавах 2 n , где n - число октав.

    Если полоса в одну октаву слишком широка, то можно применять интервалы с меньшим отношением частот в пол-октавы или трети октавы.

    Средняя частота октавы (полуоктава) не равна среднему арифметическому от нижней и верхней частот октавы, а равна 0,707 f В .

    Частоты, найденные подобным образом, называют среднеквадратичными.

    Для двух соседних октав средние частоты также образуют октавы. Пользуясь этим свойством, можно по желанию один и тот же логарифмический ряд частот считать либо границами октав, либо их средними частотами.

    На бланках с логарифмической сеткой средняя частота делит октавный ряд пополам.

    На оси частот в логарифмическом масштабе на каждую треть октавы приходятся равные отрезки оси, каждый длиной в одну треть октавы.

    При испытаниях электроакустической аппаратуры и проведении акустических измерений рекомендуется применять ряд предпочтительных частот. Частоты этого ряда являются членами геометрической прогрессии со знаменателем 1,122. Для удобства значения некоторых частот округлены в пределах ±1%.

    Интервал между рекомендованными частотами составляет одну шестую октавы. Сделано это не случайно: ряд содержит достаточно большой набор частот для разных видов измерений и вбирает ряды частот с интервалами в 1/3, 1/2 и целую октаву.

    И еще одно важное свойство ряда предпочтительных частот. В некоторых случаях в качестве основного интервала частот используется не октава, а декада. Так вот, предпочтительный ряд частот в равной мере можно рассматривать и как двоичный (октавный), и как десятичный (декадный).

    Знаменатель прогрессии, на основе которой построен предпочтительный ряд частот, численно равен 1дБ напряжения, или 1/2 дБ мощности.

    ПРЕДСТАВЛЕНИЕ ИМЕНОВАННЫХ ЧИСЕЛ В ДЕЦИБЕЛАХ

    До сих пор мы полагали, что и делимое и делитель под знаком логарифма имеют произвольную величину и для выполнения децибельного пересчета важно знать только их отношение независимо от абсолютных значений.

    В децибелах можно выражать также конкретные значения мощностей, а также напряжений и токов. Когда величина одного из членов, стоящих под знаком логарифма в рассмотренных ранее формулах задана, второй член отношения и числа децибел будут однозначно определять друг друга. Следовательно, если задаться какой-либо эталонной мощностью (напряжением, током) в качестве условного уровня сравнения, то другой мощности (напряжению, току), сопоставляемой с ней, будет соответствовать строго определенное число децибел. Нулю децибел в этом случае отвечает мощность, равная мощности условного уровня сравнения, так как при N P = 0 Р 2 1 поэтому этот уровень обычно называют нулевым. Очевидно, что при разных нулевых уровнях одна и та же конкретная мощность (напряжение, ток) будут выражаться разными числами децибел.

    где Р - мощность, подлежащая преобразованию в децибелы, а Р 0 - нулевой уровень мощности. Величина Р 0 ставится в знаменателе, при этом положительными децибелами выражаются мощности Р > Р 0 .

    Условный уровень мощности, с которым производится сравнение, в принципе может быть любым, однако не каждый был бы удобен для практического использования. Чаще всего за нулевой уровень выбирается мощность в 1 мВт, рассеиваемая на резисторе сопротивлением 600 Ом. Выбор этих параметров произошел исторически: первоначально децибел как единица измерения появился в технике телефонной связи. Волновое сопротивление воздушных двухпроводных линий из меди близко к 600 Ом, а мощность в 1 мВт развивает без усиления высококачественный угольный телефонный микрофон на согласованном сопротивлении нагрузки.

    Для случая, когда Р 0 = 1 мВт=10 –3 Вт: P р = 10 lg P + 30

    Тот факт, что децибелы представляемого параметра отчитываются относительно определенного уровня, подчеркивают термином «уровень»: уровень помех, уровень мощности, уровень громкости

    Пользуясь этой формулой, легко найти, что относительно нулевого уровня 1 мВт мощность 1 Вт определяется как 30 дБ, 1 кВт как 60 дБ, а 1 МВт - это 90 дБ, т. е. практически все мощности, с которыми приходится встречаться, укладываются в пределах первой сотни децибел. Мощности, меньшие 1 мВт, будут выражаться отрицательными числами децибел.

    Децибелы, определенные относительно уровня 1 мВт, называют децибел-милливаттом и обозначают дБм или dBm. Наиболее распространенные значения нулевых уровней сведены в таблицу 3.

    Аналогичным образом можно представить формулы для выражения в децибелах напряжений и токов:

    где U и I - напряжение или ток, подлежащие преобразованию, a U 0 и I 0 - нулевые уровни этих параметров.

    Тот факт, что децибелы представляемого параметра отчитываются относительно определенного уровня, подчеркивают термином «уровень»: уровень помех, уровень мощности, уровень громкости.

    Чувствительность микрофонов , т. е. отношение выходного электрического сигнала к звуковому давлению, действующему на диафрагму, часто выражают в децибелах, сравнивая мощность, развиваемую микрофоном на номинальном нагрузочном сопротивлении, со стандартным нулевым уровнем мощности P 0 =1 мВт . Этот параметр микрофона носит название стандартного уровня чувствительности микрофона . Типовыми условиями испытания принято считать звуковое давление 1 Па частотой 1 кГц, нагрузочное сопротивление для динамического микрофона - 250 Ом.

    Таблица 3. Нулевые уровни для измерения именованных чисел

    Обозначение Описание
    междунар. русское
    dBс дБн опорным является уровень несущей частоты (англ. carrier) или основной гармоники в спектре; например, «уровень искажений составляет –60 дБн».
    dBu дБu опорное напряжение 0,775 В, соответствующее мощности 1 мВт на нагрузке 600 Ом; например, стандартизованный уровень сигнала для профессионального аудио оборудования составляет +4 дБu, то есть 1,23 В.
    dBV дБВ опорное напряжение 1 В на номинальной нагрузке (для бытовой техники обычно 47 кОм); например, стандартизованный уровень сигнала для бытового аудио оборудования составляет –10 дБВ, то есть 0,316 В
    dBμV дБмкВ опорное напряжение 1мкВ; например, «чувствительность приёмника составляет –10дБмкВ».
    dBm дБм опорная мощность 1мВт, соответствующая мощности 1 милливатт на номинальной нагрузке (в телефонии 600 Ом, для профессиональной техники обычно 10 кОм для частот менее 10МГц, 50 Ом для высокочастотных сигналов, 75 Ом для телевизионных сигналов); например, «чувствительность сотового телефона составляет –110 дБм»
    dBm0 дБм0 опорная мощность в дБм в точке нулевого относительного уровня. dBm - опорное напряжение соответствует тепловому шуму идеального резистора сопротивлением 50 Ом при комнатной температуре в полосе 1 Гц. Например, «уровень шума усилителя составляет 6 дБм0»
    dBFS
    (англ. Full Scale - «полная шкала») опорное напряжение соответствует полной шкале прибора; например, «уровень записи составляет –6 dBfs»
    dBSPL
    (англ. Sound Pressure Level - «уровень звукового давления») - опорное звуковое давление 20 мкПа, соответствующее порогу слышимости; например, «громкость 100 dBSPL».
    dBPa - опорное звуковое давление 1 Па или 94 дБ звуковой шкалы громкости dBSPL; например, «для громкости 6 dBPa микшером установили +4 dBu, а регулятором записи –3 dBFS, искажения при этом составили –70 dBc».
    dBA, dBB,
    dBC, dBD

    опорные уровни выбраны в соответствии с частотными характеристиками стандартных «весовых фильтров» типа A, B, C или D cоответственно (фильтры отражают кривые равной громкости для разных условий, см. ниже в разделе «Шумомеры»)

    Мощность, развиваемая динамическим микрофоном, естественно, чрезвычайно мала, гораздо меньше 1 мВт, и уровень чувствительности микрофона поэтому выражается отрицательными децибелами. Зная стандартный уровень чувствительности микрофона (он приводится в паспортных данных), можно вычислить его чувствительность в единицах напряжения.

    В последние годы для характеристики электрических параметров радиоаппаратуры стали применять в качестве нулевых уровней и другие величины, в частности 1 пВт, 1 мкВ, 1 мкВ/м (последний - для оценки напряженности поля).

    Иногда возникает необходимость пересчитать известный уровень мощности P Р или напряжения P U , заданные относительно одного нулевого уровня Р 01 (или U 01 ) на другой Р 02 (или U 02 ). Сделать это можно по следующей формуле:

    Возможность представления в децибелах как отвлеченных, так и именованных чисел приводит к тому, что одно и то же устройство может характеризоваться разными числами децибел. Эту двойственность децибел надо иметь в виду. Защитой от ошибок тут может служить ясное понимание природы определяемого параметра.

    Во избежание путаницы желательно указывать опорный уровень явно, например –20 дБ (относительно 0.775 B).

    При пересчёте уровней мощностей в уровни напряжений и обратно надо обязательно учитывать сопротивление, являющиеся стандартным для данной задачи. В частности, дБВ для 75-омной ТВ-цепи соответствует (дБм–11дБ); дБмкВ для 75-омной ТВ-цепи соответствует (дБм+109дБ).

    ДЕЦИБЕЛЫ В АКУСТИКЕ

    До сих пор, говоря о децибелах, мы оперировали электрическими терминами - мощностью, напряжением, током, сопротивлением. Между тем логарифмические единицы широко применяют и в акустике, где они являются наиболее часто применяемой единицей при количественных оценках звуковых величин.

    Звуковое давление р представляет избыточное давление в среде по отношению к постоянному давлению, существующему там до появления звуковых волн (единица измерения - паскаль (Па)).

    Примером приемников звукового давления (или градиента звукового давления) может служить большинство типов современных микрофонов, которые преобразуют это давление в пропорциональные электрические сигналы.

    Интенсивность звука связана со звуковым давлением и колебательной скоростью частиц воздуха простой зависимостью:

    J=pv

    Если звуковая волна распространяется в свободном пространстве, где нет отражения звука, то

    v=p/(ρc)

    здесь ρ - плотность среды, кг/м3; с - скорость звука в среде, м/с. Произведение ρc характеризует среду, в которой происходит распространение звуковой энергии, и называется ее удельным акустическим сопротивлением . Для воздуха при нормальном атмосферном давлении и температуре 20° С ρc =420 кг/м2*с; для воды ρc = 1,5*106 кг/м2*с.

    Можно записать, что:

    J=р 2 / (ρс)

    все, что говорилось о преобразовании в децибелы электрических величин, в равной мере относится и к акустическим явлениям

    Если сопоставить эти формулы с формулами, выведенными ранее для мощности. тока, напряжения и сопротивления, то легко обнаружить аналогию между отдельными понятиями, характеризующими электрические и акустические явления, и уравнениями, описывающими количественные зависимости между ними.

    Таблица 4. Связь между электрическими и акустическими характеристиками

    Аналогом электрической мощности являются акустическая мощность и интенсивность звука; аналогом напряжения служит звуковое давление; электрический ток соответствует колебательной скорости, а электрическое сопротивление - удельному акустическому сопротивлению. По аналогии с законом Ома для электрической цепи можно говорить об акустическом законе Ома. Следовательно, все, что говорилось о преобразовании в децибелы электрических величин, в равной мере относится и к акустическим явлениям.

    Применение децибел в акустике очень удобно. Интенсивности звуков, с которыми приходится иметь дело в современных условиях, могут различаться в сотни миллионов раз. Такой огромный диапазон изменений акустических величин создает большие неудобства при сопоставлении их абсолютных значений, а при использовании логарифмических единиц эта проблема снимается. Кроме того, установлено, что громкость звука при оценке ее на слух возрастает примерно пропорционально логарифму интенсивности звука. Таким образом, уровни этих величин, выраженные в децибелах, довольно близко соответствуют громкости, воспринимаемой ухом. Для большинства людей с нормальным слухом изменение громкости звука частотой 1 кГц ощущается при изменении интенсивности звука примерно на 26%, т. е. на 1 дБ.

    В акустике по аналогии с электротехникой определение децибел базируется на отношении двух мощностей:

    где J 2 и J 1 - акустические мощности двух произвольных источников звука.

    Подобным же образом в децибелах выражается отношение двух интенсивностей звука:

    Последнее уравнение справедливо только при условии равенства акустических сопротивлений, другими словами, постоянства физических параметров среды, в которой распространяются звуковые волны.

    Децибелы, определенные по приведенным выше формулам, не связаны с абсолютными значениями акустических величин и применяются для оценки затухания звука, например эффективности звуковой изоляции и систем подавления и заглушения шумов. Подобным образом выражаются и неравномерности частотных характеристик, т. е. разность максимального и минимального значений в заданном диапазоне частот различных излучателей и приемников звука: микрофонов, громкоговорителей и пр. Отсчет при этом обычно ведется от среднего значения рассматриваемой величины, либо (при работе в звуковом диапазоне) относительно значения при частоте 1 кГц.

    В практике акустических измерений, однако, как правило, приходится иметь дело со звуками, значения которых должны быть выражены конкретными числами. Аппаратура для проведения акустических измерений сложнее аппаратуры для электрических измерений, а по точности существенно уступает ей. С целью упрощения техники измерений и снижения погрешности в акустике отдается предпочтение измерениям относительно эталонных, калиброванных уровней, величины которых известны. С этой же целью для измерения и исследования акустических сигналов их преобразуют в электрические.

    Абсолютные значения мощностей, интенсивностей звуков и звуковых давлений также могут быть выражены в децибелах, если в приведенных выше формулах задаваться значениями одного из членов под знаком логарифма. Международным соглашением уровнем отсчета интенсивности звука (нулевым уровнем) принято считать J 0 = 10 –12 Вт/м 2 . Эту ничтожную интенсивность, под действием которой амплитуда колебаний барабанной перепонки меньше размеров атома, условно принято считать порогом слышимости уха в области частот наибольшей чувствительности слуха. Ясно, что все слышимые звуки выражаются относительно этого уровня только положительными децибелами. Фактический порог слышимости для людей с нормальным слухом немного выше и равен 5-10 дБ.

    Для представления интенсивности звука в децибелах относительно заданного уровня используют формулу:

    Значение интенсивности, вычисленное по этой формуле, принято называть уровнем интенсивности звука .

    Подобным образом можно выразить и уровень звукового давления:

    Чтобы уровни интенсивности звука и звукового давления в децибелах численно выражались одной величиной, в качестве нулевого уровня звукового давления (порога звукового давления) должно быть принято значение:

    Пример. Определим, какой уровень интенсивности в децибелах создает оркестр со звуковой мощностью 10 Вт на расстоянии r = 15 м.

    Интенсивность звука на расстоянии r = 15 м от источника составит:

    Уровень интенсивности в децибелах:

    Тот же результат будет получен, если преобразовать в децибелы не уровень интенсивности, а уровень звукового давления.

    Так как в месте приема звука уровень интенсивности звука и уровень звукового давления выражаются одинаковым числом децибел, на практике часто применяется термин «уровень в децибелах» без указания, к какому именно параметру эти децибелы относятся.

    Определив уровень интенсивности в децибелах в какой-либо точке пространства на расстоянии r 1 от источника звука (расчетным или опытным путем), нетрудно вычислить уровень интенсивности на расстоянии r 2 :

    Если на приемник звука одновременно воздействуют два или несколько источников звука и известна интенсивность звука в децибелах, создаваемая каждым из них, то для определения результирующей величины децибелы следует обратить в абсолютные значения интенсивности (Вт/м2), сложить их, и эту сумму снова преобразовать в децибелы. Складывать сразу децибелы в этом случае нельзя, так как это соответствовало бы произведению абсолютных значений интенсивностей.

    Если имеется n несколько одинаковых источников звука с уровнем каждого L J , то их суммарный уровень будет:

    Если уровень интенсивности одного источника звука превышает уровни остальных на 8-10 дБ и более, можно учитывать только один этот источник, а действием остальных пренебречь.

    Помимо рассмотренных акустических, уровней иногда можно встретить и понятие уровня звуковой мощности источника звука, определяемого по формуле:

    где Р - звуковая мощность характеризуемого произвольного источника звука, Вт; Р 0 - начальная (пороговая) звуковая мощность, величина которой берется обычно равной P 0 =10 –12 Вт.

    УРОВНИ ГРОМКОСТИ

    Чувствительность уха к звукам разных частот различна. Зависимость эта довольно сложна. При небольших уровнях интенсивности звука (примерно до 70 дБ) максимальная чувствительность составляет 2-5 кГц и убывает с повышением и понижением частоты. Поэтому звуки одинаковой интенсивности, но разных частот будут казаться на слух разными по громкости. С ростом силы звука частотная характеристика уха выравнивается и при больших уровнях интенсивности (80 дБ и выше) ухо реагирует приблизительно одинаково на звуки разных частот звукового диапазона. Из этого следует, что интенсивность звука, которая измеряется специальными широкополосными приборами, и громкость, которая фиксируется ухом, - понятия не равнозначные.

    Уровень громкости звука любой частоты характеризуется величиной уровня равного по громкости звука частотой 1 кГц

    Уровень громкости звука любой частоты характеризуется величиной уровня равного по громкости звука частотой 1 кГц. Уровни громкости характеризуются так называемыми кривыми равных громкостей, каждая из которых показывает, какой уровень интенсивности на разных частотах должен развить источник звука, чтобы создать впечатление равной громкости с тоном 1 кГц заданной интенсивности (рис. 4).


    Рис. 4. Кривые равной громкости

    Кривые равной громкости представляют по существу семейство частотных характеристик уха в децибельном масштабе для разных уровней интенсивности. Отличие их от обычных АЧХ состоит лишь в способе построения: «завал» характеристики, т. е. снижение коэффициента передачи, здесь изображен повышением, а не понижением соответствующего участка кривой.

    Единице, характеризующей уровень громкости, во избежание путаницы с децибелами интенсивности и звукового давления присвоено особое наименование - фон .

    Уровень громкости звука в фонах численно равен уровню звукового давления в децибелах чистого тона с частотой 1 кГц, равного с ним по громкости.

    Другими словами, один фон - это 1 дБ звукового давления тона частотой 1 кГц с поправкой на частотную характеристику уха. Между двумя, этими единицами нет постоянного соотношения: оно меняется в зависимости от уровня громкости сигнала и его частоты. Только для токов частотой 1 кГц численные значения для уровня громкости в фонах и уровня интенсивности в децибелах совпадают.

    Если обратиться к рис. 4 и проследить ход одной из кривых, например, для уровня 60 фон, то нетрудно определить, что для обеспечения равной громкости с тоном 1 кГц на частоте 63 Гц требуется интенсивность звука 75 дБ, а на частоте 125 Гц только 65 дБ.

    В высококачественных усилителях звуковой частоты применяются ручные регуляторы громкости с тонкомпенсацией, или, как их еще называют, компенсированные регуляторы. Такие регуляторы одновременно с регулировкой величины входного сигнала в сторону уменьшения обеспечивают подъем частотной характеристики в области низших частот, благодаря чему для слуха создается неизменный тембр звучания при различных громкостях воспроизведения звука.

    Исследованиями установлено также, что изменение громкости звука вдвое (по оценке на слух) примерно эквивалентно изменению уровня громкости на 10 фон. Эта зависимость положена в основу оценки громкости звука. За единицу громкости, называемую сон , условно принят уровень громкости 40 фон. Удвоенной громкости, равной двум сон, соответствует 50 фон, четырем сон - 60 фон и т. д. Пересчет уровней громкости в единицы громкости облегчается графиком на рис. 5.


    Рис. 5. Связь между громкостью и уровнем громкости

    Большинство звуков, с которыми приходится иметь дело в повседневной жизни, имеют шумовой характер. Характеристика громкости шумов на основе сопоставления с чистыми тонами 1 кГц проста, но приводит к тому, что оценка шума на слух может расходиться с показаниями измерительных приборов. Объясняется это тем, что при равных уровнях громкости шума (в фонах) наиболее раздражающее действие на человека оказывают составляющие шума в диапазоне 3-5 кГц. Шумы могут восприниматься как равно неприятные, хотя их уровни громкости не равны.

    Раздражающее действие шума более точно оценивается другим параметром, так называемым уровнем воспринимаемого шума . Мерой воспринимаемого шума служит уровень звука равномерного шума в октавной полосе со средней частотой 1 кГц, который в заданных условиях оценивается слушателем как одинаково неприятный с измеряемым шумом. Уровни воспринимаемого шума характеризуются единицами PNdB или РNдБ. Расчет их ведется по специальной методике.

    Дальнейшим развитием системы оценки шумов являются так называемые эффективные уровни воспринимаемого шума, выражаемые в ЕРNдБ. Система ЕРNдБ позволяет комплексно оценивать характер воздействующего шума: частотный состав, дискретные составляющие в его спектре, а также продолжительность шумового воздействия.

    По аналогии с единицей громкости сон введена единица шумности - ной .

    За один ной принята шумность равномерного шума в полосе 910-1090 Гц при уровне звукового давления 40 дБ. В остальном нои сходны с сонами: рост шумности вдвое соответствует росту уровня воспринимаемого шума на 10 РNдБ, т. е. 2 ной = 50 РNдБ, 4 ной = 60 РNдБ и т. д.

    Работая с акустическими понятиями, следует иметь в виду, что интенсивность звука представляет объективное физическое явление, которое может быть точно определено и измерено. Оно реально существует независимо от того, слышит его кто-нибудь или нет. Громкость звука определяет эффект, который звук производит на слушателя, и является, поэтому, чисто субъективным понятием, так как зависит от состояния органов слуха человека и его личных свойств к восприятию звука.

    ШУМОМЕРЫ

    Для измерения всевозможных шумовых характеристик применяют специальные приборы - шумомеры. Шумомер представляет автономный переносный прибор, позволяющий измерять непосредственно в децибелах уровни интенсивности звука в широких пределах относительно стандартных уровней.

    Шумомер (рис. 6) состоит из высококачественного микрофона, широкополосного усилителя, переключателя чувствительности, меняющего усиление ступенями по 10 дБ, переключателя частотных характеристик и графического индикатора, который обычно обеспечивает несколько вариантов представления измеряемых данных - от цифр и таблицы до графика.


    Рис. 6. Портативный цифровой шумомер

    Современные шумомеры весьма компактны, что позволяет производить измерения и в труднодоступных местах. Из отечественных шумомеров можно назвать прибор компании «Октава-Электродизайн» «Октава-110А» (http://www.octava.info/?q=catalog/soundvibro/slm) .

    Шумомеры позволяют определять как общие уровни интенсивностей звука при измерениях с линейной частотной характеристикой, так и уровни громкости звука в фонах при измерениях с частотными характеристиками, сходными с характеристиками человеческого уха. Диапазон измерений уровней звуковых давлений находится обычно в пределах от 20-30 до 130-140 дБ относительно стандартного уровня звукового давления 2*10–5 Па. С помощью сменных микрофонов уровень измерений может быть расширен до 180 дБ.

    В зависимости от метрологических параметров и технических характеристик отечественные шумомеры подразделяются на первый и второй классы.

    Частотные характеристики всего тракта шумомера, включая микрофон, стандартизированы. Всего имеется пять частотных характеристик. Одна из них линейна в пределах всего рабочего диапазона частот (условное обозначение Лин ), четыре другие приближенно повторяют характеристики уха человека для чистых тонов при разных уровнях громкости. Они названы первыми буквами латинского алфавита А, В, С и D . Вид этих характеристик показан на рис. 7. Переключатель частотных характеристик не зависит от переключателя пределов измерений. Для шумомеров первого класса обязательны характеристики А, В, С и Лин . Частотная характеристика D - дополнительная. Шумомеры второго класса должны иметь характеристики А и С ; применение остальных допускается.


    Рис. 7. Стандартные частотные характеристики шумомеров

    Характеристика А имитирует ухо примерно на уровне 40 фон. Эта характеристика используется при измерении слабых шумов - до 55 дБ и при замерах уровней громкости. В практических условиях чаще всего пользуются частотной характеристикой с коррекцией А . Объясняется это тем, что, хотя восприятие звука человеком гораздо сложнее простой частотной зависимости, определяющей характеристику А , во многих случаях результаты измерений прибором хорошо согласуются с оценкой шума на слух при небольших уровнях громкости. Многими стандартами - отечественными и зарубежными - оценку шумов рекомендуется проводить по характеристике А независимо от фактического уровня интенсивности звука.

    Характеристика В повторяет характеристику уха на уровне 70 фон. Она применяется при измерении шумов в пределах 55-85 дБ.

    Характеристика С равномерна в диапазоне 40-8000 Гц. Этой характеристикой пользуются при измерении значительных уровней громкости - от 85 фон и выше, при измерениях уровней звукового давления - независимо от пределов измерения, а также при подключениях к шумомеру устройств для измерения спектрального состава шума в тех случаях, когда шумомер не имеет частотной характеристики Лин .

    Характеристика D - вспомогательная. Она представляет усредненную характеристику уха примерно на уровне 80 фон с учетом повышения его чувствительности в полосе от 1,5 до 8 кГц. При пользовании этой характеристикой показания шумомера более точно, чем по другим характеристикам, соответствуют уровню воспринимаемого шума человеком. Эта характеристика применяется главным образом при оценке раздражающего действия шума большой интенсивности (самолетов, быстроходных машин и т. п.).

    В составе шумомера имеется также переключатель Быстро - Медленно - Импульс , управляющий временными характеристиками прибора. Когда переключатель установлен в положение Быстро , прибор успевает следить за быстрыми изменениями уровней звука, в положении Медленно прибор показывает среднее значение измеряемого шума. Временная характеристика Импульс применяется при регистрации коротких звуковых импульсов. Некоторые типы шумомеров содержат также интегратор с постоянной времени 35 мс, имитирующий инерционность звуковосприятия человека.

    При пользовании шумомером результаты измерений будут различаться в зависимости от установленной частотной характеристики. Поэтому при записи показаний для исключения путаницы указывается и вид характеристики, при которой производились измерения: дБ (А ), дБ (В ), дБ (С ) или дБ (D ).

    Для калибровки всего тракта микрофон - измеритель в комплект шумомера обычно входит акустический калибратор, назначение которого - создавать равномерный шум определенного уровня.

    Согласно действующей в настоящее время инструкции «Санитарные нормы допустимого шума в помещениях жилых и общественных зданий и на территории жилой застройки» нормируемыми параметрами постоянного или прерывистого шума являются уровни звуковых давлений (в децибелах) в октавных полосах частот со средними частотами 63, 125, 250, 500, 1000, 2000, 4000, 8000 Гц. Для непостоянного шума, например шума от проезжающего транспорта, нормируемым параметром является уровень звука в дБ(А ).

    Установлены следующие суммарные уровни звука, измеренные по шкале А шумомера: жилые помещения - 30 дБ, аудитории и классы учебных заведений - 40 дБ, территории жилой застройки и площадки отдыха - 45 дБ, рабочие помещения административных зданий - 50 дБ (А ).

    Для санитарной оценки уровня шума в показания шумомера вносятся поправки от –5 дБ до +10 дБ, которые учитывают характер шума, суммарное время его действия, время суток и месторасположение объекта. Например, в дневное время норма допустимого шума в жилых помещениях с учетом поправки составляет 40 дБ.

    В зависимости от спектрального состава шума ориентировочная норма предельно допустимых уровней, дБ, характеризуется следующими цифрами:

    Высокочастотный от 800 Гц и выше 75-85
    Среднечастотный 300-800 Гц 85-90
    Низкочастотный ниже 300 Гц 90-100

    При отсутствии шумомера ориентировочную оценку уровней громкости различных шумов можно проводить с помощью таблицы. 5.

    Таблица 5. Шумы и их оценка

    Оценка громкости
    на слух
    Уровень
    шума, дБ
    Источник и место измерения шума
    Оглушительный 160 Повреждение барабанной перепонки.
    140-170 Реактивные двигатели (вблизи).
    140 Предел терпимости к шуму.
    130 Болевой порог (звук воспринимается как боль); поршневые авиадвигатели(2-3 м).
    120 Гром над головой.
    110 Быстроходные мощные двигатели (2-3 м); клепальная машина (2-3 м); очень шумный цех.
    Очень громкий 100 Симфонический оркестр (пики громкости); деревообрабатывающие станки (на рабочем месте)
    90 Уличный громкоговоритель; шумная улица; металлорежущие станки (на рабочем месте).
    80 Радиоприемник громко (2 м)
    Громкий 70 Салон автобуса; крик; свисток милиционера (15 м); улица средней шумности; шумный офис; зал большого магазина
    Умеренный 60 Спокойный разговор (1 м).
    50 Легковая машина (10-15 м); спокойный офис; жилое помещение.
    Слабый 40 Шепот; читальный зал.
    60 Шелест бумаги.
    20 Больничная палата.
    Очень слабый
    10 Тихий сад; студия радиоцентра.
    0 Порог слышимости
    1 А. Белл - американский учёный, изобретатель и бизнесмен шотландского происхождения, основоположник телефонии, основатель компании Bell Telephone Company, определившей развитие телекоммуникационной отрасли в США.
    2 Логарифмы отрицательных чисел являются комплексными числами и далее рассматриваться не будут.

    И т. п., поэтому отношение D F {\displaystyle D_{F}} двух значений силовой величины F {\displaystyle F}

    D F = 20 lg ⁡ F 1 F 0 . {\displaystyle D_{F}=20\lg {\frac {F_{1}}{F_{0}}}.}

    Отсюда следует, что увеличение силовой величины на 1 дБ означает её увеличение в 10 0 , 05 {\displaystyle 10^{0,05}} ≈ 1,122 раза.

    Децибел относится к единицам, не входящим в Международную систему единиц (СИ) , но в соответствии с решением Международного комитета мер и весов допускается к применению без ограничений совместно с единицами СИ . В основном применяется в электросвязи , акустике , радиотехнике .

    Энциклопедичный YouTube

      1 / 2

      ✪ Что такое децибел

      ✪ EdEra: Що таке децибел?

    Субтитры

    История

    Распространение децибела берёт начало от методов, используемых для количественной оценки потери (ослабления) сигнала в телеграфных и телефонных линиях. Единицей потерь изначально была миля стандартного кабеля (англ. mile of standard cable - m.s.c.). 1 m.s.c. - это отношение мощностей сигнала с частотой 800 Гц на двух концах кабеля длиной в 1 милю (примерно 1,6 км), имеющего распределённое сопротивление 88 Ом (на петлю) и распределённую ёмкость 0,054 мкФ . Такое отношение мощностей, преобразованных в звуковые колебания, было близким к наименьшей различимой средним слушателем разнице двух сигналов по громкости. Однако миля стандартного кабеля была частотно-зависимой, и она не могла быть полноценной единицей отношения мощностей .

    Определение

    Децибелы принято использовать для измерения или выражения отношения одноимённых энергетических величин, таких как мощность, энергия, интенсивность, плотность потока мощности, спектральная плотность мощности и т. п., а также силовых величин, таких как напряжение, сила тока, напряженность поля, звуковое давление и т. п. Часто в качестве одной из величин отношения (в знаменателе) выступает общепринятая исходная (или опорная) величина. Тогда отношение, выраженное в децибелах, принято называть уровнем соответствующей физической величины (например, уровень мощности, уровень напряжения и т. д.) .

    Энергетические величины

    Примеры соотношений
    с энергетическими и силовыми величинами
    D {\displaystyle D} P 1 / P 0 {\displaystyle P_{1}/P_{0}} F 1 / F 0 {\displaystyle F_{1}/F_{0}}
    40 dB 10000 100
    20 dB 100 10
    10 dB 10 ≈ 3,16
    6 dB ≈ 4 ≈ 2
    3 dB ≈ 2 ≈ 1,41
    1 dB ≈ 1,26 ≈ 1,12
    0 dB 1 1
    −1 dB ≈ 0,79 ≈ 0,89
    −3 dB ≈ 0,5 ≈ 0,71
    −6 dB ≈ 0,25 ≈ 0,5
    −10 dB 0,1 ≈ 0,32
    −20 dB 0,01 0,1
    −40 dB 0,0001 0,01

    Отношение D P {\displaystyle D_{P}} двух значений энергетической величины P {\displaystyle P} и P 0 {\displaystyle P_{0}} , выраженное в децибелах, определяется по формуле:

    D P = 10 lg ⁡ P 1 P 0 . {\displaystyle D_{P}=10\lg {\frac {P_{1}}{P_{0}}}.} P 1 P 0 = 10 0 , 1 D P {\displaystyle {\frac {P_{1}}{P_{0}}}=10^{0,1D_{P}}} 00 или00 P 1 = P 0 ⋅ 10 0 , 1 D P . {\displaystyle P_{1}=P_{0}\cdot 10^{0,1D_{P}}.}

    Силовые величины

    Энергетические величины пропорциональны квадратам силовых величин. Например, в электрической цепи мощность P {\displaystyle P} , рассеиваемая в тепло на нагрузке с сопротивлением R {\displaystyle R} при напряжении U {\displaystyle U} , определяется по формуле:

    P = U 2 R . {\displaystyle P={U^{2} \over R}.}

    Отсюда отношение двух величин:

    P 1 P 0 = U 1 2 R 1 R 0 U 0 2 . {\displaystyle {P_{1} \over P_{0}}={U_{1}^{2} \over R_{1}}{R_{0} \over U_{0}^{2}}.}

    Логарифмическое отношение в частном случае, при R 1 = R 0 {\displaystyle R_{1}=R_{0}} :

    10 lg ⁡ P 1 P 0 = 10 lg ⁡ (U 1 U 0) 2 = 20 lg ⁡ U 1 U 0 . {\displaystyle 10\lg {P_{1} \over P_{0}}=10\lg {\left({U_{1} \over U_{0}}\right)}^{2}=20\lg {U_{1} \over U_{0}}.}

    Таким образом, сохранение численных значений в децибелах при переходе от отношения мощностей к отношению напряжений при одинаковых нагрузках требует, чтобы выполнялось следующее соотношение:

    D P = D U , {\displaystyle D_{P}=D_{U},} 00 где0 D U = 20 lg ⁡ U 1 U 0 . {\displaystyle D_{U}=20\lg {U_{1} \over U_{0}}.} U 1 U 0 = 10 0 , 05 D U {\displaystyle {\frac {U_{1}}{U_{0}}}=10^{0,05D_{U}}} 00 или00 U 1 = U 0 ⋅ 10 0 , 05 D U . {\displaystyle U_{1}=U_{0}\cdot 10^{0,05D_{U}}.}

    Определение единицы бел

    Бел (русское обозначение: Б; международное: B) выражает отношение двух мощностей как десятичный логарифм этого отношения .

    Сравнение логарифмических единиц

    Единица Обозначение Изменение энергетической
    величины в … раз
    Изменение силовой
    величины в … раз
    Пересчёт в …
    дБ Б Нп
    децибел дБ, dB 10 10 {\displaystyle {\sqrt[{10}]{10}}} ≈ 1,259 10 20 {\displaystyle {\sqrt[{20}]{10}}} ≈ 1,122 1 0,1 ≈0,1151
    бел Б, B 10 10 {\displaystyle {\sqrt {10}}} ≈ 3,162 10 1 ≈1,151
    непер Нп, Np e 2 ≈ 7,389 e ≈ 2,718 ≈8,686 ≈0,8686 1

    Применение

    Децибелы широко применяются в областях техники, где требуется измерение или представление величин, меняющихся в широком диапазоне: в радиотехнике, антенной технике, в системах передачи информации, автоматического регулирования и управления, в оптике, акустике (в децибелах измеряется уровень громкости звука) и др. Так, в децибелах принято измерять или указывать динамический диапазон (например, диапазон громкости звучания музыкального инструмента), затухание волны при распространении в поглощающей среде, коэффициент затухания радиочастотного кабеля, коэффициент усиления и коэффициент шума усилителя.

    Акустика

    Звуковое давление - силовая величина, а интенсивность звука , пропорциональная квадрату звукового давления, - энергетическая величина. Например, если громкость звука (субъективно определяемая его интенсивностью) возросла на 10 дБ, то это значит, что интенсивность звука возросла в 10 раз, а звуковое давление - приблизительно в 3,16 раза.

    Использование децибелов при указании громкости звука обусловлено человеческой способностью воспринимать звук в очень большом диапазоне изменений его интенсивности. Применение линейной шкалы оказывается практически неудобным. Кроме того, на основании закона Вебера - Фехнера , ощущение громкости звука пропорционально логарифму его интенсивности. Отсюда удобство логарифмической шкалы. Диапазон величин звукового давления от минимального порога слышимости звука человеком (20 мкПа) до максимального, вызывающего болевые ощущения, составляет примерно 120 дБ. Например, утверждение «громкость звука составляет 30 дБ» означает, что интенсивность звука в 1000 раз превышает порог слышимости звука человеком.

    Для выражения громкости звука также используют единицы фон и сон , учитывающие частотную и субъективную восприимчивость звука человеком.

    Удобства применения децибелов

    Прежде всего следует отметить удобство децибела по сравнению с единицей бел . Для практических применений бел оказался слишком крупной единицей, часто предполагающей дробную запись значения логарифмической величины. Перечисленные ниже удобства так или иначе связаны с применением не только децибелов, а логарифмической шкалы и логарифмических величин вообще.

    • Характер отображения в органах чувств человека и животных изменений течения многих физических и биологических процессов пропорционален не амплитуде входного воздействия, а логарифму входного воздействия (см. Закон Вебера - Фехнера). Эта особенность делает применение логарифмических шкал, логарифмических величин и их единиц вполне естественным. Например, одной из таких шкал является музыкальная равномерно темперированная шкала частот.
    • Логарифмическая шкала даёт наглядное графическое представление и упрощение анализа величины, изменяющейся в очень широких пределах (примеры - диаграмма направленности антенны, амплитудно-частотная характеристика (АЧХ) системы автоматического регулирования). Это же относится к передаточным частотным характеристикам электрических фильтров (см. логарифмическая амплитудно-фазовая частотная характеристика). При этом форма кривой упрощается и возможно применение кусочно-линейной аппроксимации, при которой скорость убывания частотной характеристики имеет размерность дБ/декада или дБ/октава. Упрощается анализ частотной характеристики фильтров, составленных из последовательно включенных звеньев с независимыми друг от друга частотными характеристиками. Следует заметить, что построение графиков в логарифмическом масштабе требует определённого навыка (см. Логарифмическая бумага).
    • Логарифмическое представление некоторых относительных величин в ряде случаев упрощает математические операции с ними, в частности, умножение и деление заменяются сложением и вычитанием. Например, если собственные коэффициенты усиления последовательно включённых усилителей выражены в децибелах, то общий коэффициент усиления находится как сумма собственных коэффициентов.

    Опорные величины и обозначения уровней

    Если в качестве одной из величин отношения (в знаменателе) выступает общепринятая исходная (или опорная) величина X ref , то отношение, выраженное в децибелах, называют уровнем (иногда называют абсолютным уровнем ) соответствующей физической величины X и обозначают L X (от англ. level ).

    В соответствии с действующими стандартами , при необходимости указать исходную величину её значение помещают в скобках за обозначением логарифмической величины. Например, уровень L P звукового давления P можно записать: L P (исх. 20 мкПа) = 20 дБ, а с использованием международных обозначений - L P (re 20 µPa) = 20 dB (re - сокращение от англ. reference ). Допускается указывать значение исходной величины в скобках за значением уровня, например: 20 дБ (исх. 20 мкПа). Также используется краткая форма, например, уровень L W мощности W можно записать: L W (1 мВт) = 30 дБ, или L W = 30 дБ (1 мВт). Значение «1» исходной величины может быть опущено, например, L W = 30 дБ (мВт). То есть, если в скобках указана только размерность исходной величины, а значение величины не указано, то подразумевается, что оно равно «1». Для сокращения записи широко используются специальные обозначения, например: L W = 30 дБм. Запись означает, что уровень мощности составляет +30 дБ относительно 1 мВт, то есть мощность равна 1 Вт.

    Специальные обозначения

    Приведены некоторые специальные обозначения, которые в предельно краткой форме указывают на значение исходной (опорной) величины, по отношению к которой определён соответствующий уровень, выраженный в децибелах . Для указанных ниже опорных величин под электрическим напряжением понимается его среднеквадратичное (эффективное) значение.

    • dBW (русское дБВт ) - опорная мощность 1 Вт. Например, уровень мощности +30 дБВт соответствует мощности 1 кВт.
    • dBm (русское дБм ) - опорная мощность 1 мВт.
    • dBm0 (русское дБм0 ) - опорная мощность 1 мВт. Обозначение применяется в электросвязи для указания абсолютного уровня мощности, приведённого к так называемой точке нулевого относительного уровня.
    • dBV (русское дБВ ) - опорное напряжение 1 В.
    • dBuV или dBμV (русское дБмкВ ) - опорное напряжение 1 мкВ.
    • dBu (русское дБн ) - опорное напряжение 0 , 600 {\displaystyle {\sqrt {0,600}}} ≈ 0,775 В, соответствующее мощности 1 мВт на нагрузке 600 Ом.
    • dBrn - опорное напряжение соответствует мощности теплового шума идеального резистора с сопротивлением R {\displaystyle R} равным 50 Ом при комнатной температуре в полосе частот 1 Гц: V n o i s e = 4 k B T R = 9 ⋅ 10 − 10 [ V ] {\displaystyle V_{noise}={\sqrt {4k_{B}TR}}=9\cdot 10^{-10}\left[{\text{V}}\right]} . Это значение соответствует уровню напряжения −61 dBμV или уровню мощности −168 dBm.
    • dBFS (от англ. full scale - «полная шкала») - опорный сигнал (мощность, напряжение) соответствует полной шкале аналого-цифрового преобразователя .
    • dB SPL (от

    Слово "децибел" состоит из двух частей: приставки "деци" и корня "бел". "Деци" дословно означает "десятая часть", т.е. десятая часть "бэла". Значит, чтобы понять что такое децибел надо понять, что такое бел и всё станет на свои места.

    Давным давно Александр Белл выяснил, что человек перестает слышать звук, если мощность источника этого звука меньше, чем 10 -12 Вт/м 2 , а если она превышает 10 Вт/м 2 , то готовьте ваши ушки к неприятной боли - это болевой порог.

    Как видно разница между 10 -12 Вт/м 2 и 10 Вт/м 2 целых 13 порядков. Белл поделил расстояние между порогом слышимости и болевым порогом на 13 ступеней: от 0 (10 -12 Вт/м 2) до 13 (10 Вт/м 2). Таким образом он определил шкалу звуковой мощности.

    Тут можно сказать: "О, всё понятно!", - хорошо! Но дальше ещё интересней.

    Ближе к делу

    Мы выяснили, что децибел равен 1/10 бела, но как это применять в жизни? Приведу такой пример:

    • 0 Дб - ничего не слышно
    • 15 Дб - едва слышно (шелест листвы)
    • 50 Дб - Отчётливо слышно
    • 60 Дб - Шумно

    Да зачем это надо, если можно, к примеру, сказать: "уровень звуковой мощности 0.1 Вт/м 2 ". Дело в том, что экспериментально установлено, что человек ощущает изменение яркости, громкости и т.д. тогда, когда они изменяются логарифмически. Вот так:

    Что в белах выражается как отношение уровня измеряемого сигнала к некоторому эталонному. 1 Бэл = lg(P 1 /P 0), где P 0 - это звуковая мощность порога слышимости, ну а чтобы получить децибел надо всего-то умножить на 10: 1 Дб = 10*lg(P 1 /P 0)

    Таким образом децибел показывает логарифм отношения уровня одного сигнала к другому и используется для сравнения двух сигналов. Из формулы, кстати, видно, что децибелах можно сравнивать любые сигналы (и не только звуковую мощность), так как децибел величина безразмерная.

    Особенности

    Путаница с децибелами возникает из-за того, что существует несколько их "видов". Они условно называются амплитудными и мощностными (энергетическими).

    Формула 1 Дб = 10*lg(P 1 /P 0) - сравнивает в децибелах две энергетические величины. В данном случае мощность. А формула 1 Дб = 20*lg(A 1 /A 0) - сравнивает две амплитудные величины. К примеру, напряжение, ток и т.д.
    Перейти от амплитудных децибелов к энергетическим и обратно очень просто. Требуется просто «неэнергетические» величины преобразовать в энергетические. Покажу это на примере тока и напряжения.

    Из определения мощности P = UI = U 2 / R = I 2 * R. Подставим в 10*lg(P 1 /P 0) и после преобразования получим 20*lg(A 1 /A 0) - всё просто.

    Таким же образом будут проводится преобразования для других амплитудных значений. Подробнее как всегда можно прочитать в учебниках и справочниках.

    Зачем надо было всё усложнять?

    Понимаешь, две величины могут отличаться в миллионы раз. Таким образом простое отношение (P 1 /P 0) может давать как очень большие, так и очень маленькие значения. Согласись, что это не очень удобно в практической деятельности. Может быть это также одна из причин такой распространенности децибел (наряду со следствием из закона Вебера-Фехнера)

    Таким образом децибел позволяет от исчисления в "попугаях", т.е. в разах перейти к более конкретным и небольшим величинам. Которые можно быстро складывать и вычитать в уме. А если все же хочется оценить отношение в попугаях по известному значению в децибелах, то достаточно запомнить простое мнемоническое правило (подсмотрел у Ревича):

    Если отношение величин больше единицы, то это будет положительный Дб (+3 Дб), а если меньше - отрицательный (-3 Дб). Таким образом:

    • 3 Дб означает увеличение/уменьшение сигнала на треть
    • 6 Дб означает увеличение/уменьшение в 2 раза
    • 10 Дб соответствует изменение величины в 3 раза
    • 20 Дб соответствует изменению в 10 раз

    А теперь на примере. Пусть нам сказали, что сигнал усиливается на 50 Дб. А 50 Дб = 10 Дб + 20 Дб + 20 Дб = 3 * 10 * 10 = 300 раз. Т.е. сигнал усилился в 300 раз.

    Так что децибел всего лишь удобное инженерное соглашение, которое введено в результате некоторых практических измерений, а также выгоды от практического использования.

    При измерениях чего-то (например, напряжения) мы обычно думаем в прямых единицах (в вольтах). Но иногда более предпочтительно использовать относительную шкалу. В этом случае, наиболее часто используемой единицей измерений является децибел (дБ) - мощный инструмент, приводящий в замешательство начинающих. При знании происхождения этого термина и одного простого правила, затруднения могут быть исключены, а значение величины, выраженной в децибелах, может быть понято.

    Александр Грехэм Белл стал известен благодаря изобретению телефона. Менее известны его работы по определению порога слышимости. В 1890 году он основал Ассоциацию глухих и плохо слышащих, которая действует до сих пор. Он был первым ученым, который количественно определил чувство слуха и установил, что слуховая восприимчивость зависит не от реального уровня мощности звуковой волны, достигающей нашего уха, а от ее логарифма.

    Белл обнаружил, что порог слышимости ребенка составляет около 10 -12 Вт/м 2 , а уровень, при котором возникают болевые ощущения - около 10 Вт/м 2 . Таким образом, диапазон громкости, нормально воспринимаемой человеком, составляет 13 порядков!

    Исходя из полученных значений, Белл определил шкалу звуковой мощности от 0 до 13. Единицы громкости этой шкалы называются белами (последнее "л" от его фамилии было отброшено). Уровень звука тихого шепота составляет около 3 белов, а нормальной речи - около 6 белов.

    Поскольку ощущение громкости базируется на логарифмической шкале уровня мощности, то преобразование между мощностью и громкостью по шкале Белла выглядит следующим образом: громкость (в белах) = lg(P1/P0), где P0 - порог слышимости звука.

    Следовательно, уровень звука в 4 бела соответствует звуковой мощности, равной 10 4 P0.

    Бел стал фактически стандартной единицей измерения логарифма отношения двух энергетических уровней: отношение, выраженное в белах, есть lg(P1/P0), т.е. увеличение на 3 бела соответствует увеличению в 1000 раз. Если новое значение убывает, то логарифм отношения становится отрицательным. Чтобы сделать обратное преобразование необходимо 10 возвести в степень, равную белам.

    Важнейшая особенность белов состоит в том, что они относятся только к отношению двух мощностей или двух энергий. Если же есть необходимость описания отношения двух амплитудных сигналов, например, напряжений, то возможно лишь опираться на отношение мощностей, ассоциированных с этими напряжениями. Мощность пропорциональна квадрату напряжения или тока: V 2 и I 2 .

    Отношение двух напряжений, выраженное в белах, связано с отношением их мощностей: lg(P1/P0) = 2lg(V1/V0). Следовательно, отношение напряжений равно V1/V0 = lg10 (белы*2) .

    Стало достаточно общим выражать отношение в десятых долях бела или в децибелах (дБ). Отношение двух мощностей в дБ равняется 10lg(P1/P0), а напряжений - 10 2lg(V1/V0). Для получения отношения напряжений необходимо выполнить преобразование V1/V0 = 10 (дБ/20) .

    Порой достаточно мудрено определить, что считать амплитудной величиной, а что энергетической. Напряжение, ток, импеданс, напряженности электрического или магнитного полей и размахи любых волновых процессов считаются амплитудными величинами. Когда происходит измерение в децибелах, то вычисляется логарифм отношения квадратов этих величин. Энергия, мощность и интенсивность являются энергетическими величинами, и в отношении логарифма они используются непосредственно.

    Например, 5% напряжения одной цепи передается в другую цепь. Отношение напряжений в этом случае равно 0,05. Для измерения в децибелах необходимо взять логарифм отношения напряжений, умножить его на 2, чтобы получить отношение в белах, а затем умножить на 10 для получения отношения в дБ: 20lg(0,05) = -26 дБ связи между сигналами.

    В таблице приведены некоторые, часто используемые значения в децибелах и отношения амплитуд и мощностей.

    Радио 1967, 12

    Децибел - специфическая единица численного выражения усиления или ослабления сигнала. В децибелах оценивают коэффициенты усиления и затухания, избирательность приёмников, неравномерность частотных характеристик, интенсивность звука и многие параметры различных радиотехнических приборов, аппаратов, линий передач, антенных и других устройств. Шкалы децибел имеют многие вольтметры и авометры.

    Что же такое децибел? Прежде всего - децибел (сокращённое обозначение - дБ) не физическая величина, как, скажем, ватт, вольт, ампер, а математическое понятие. В этом отношении у децибел есть некоторое сходство с процентами. Как и проценты, децибел величина относительная и применима к оценке самых различных явлений, независимо от их природы. Но, если проценты выражают какую-то величину, отнесённую к целому, принятому за единицу, то в основе, децибела лежит более широкое понятие, характеризующее отношение двух независимых, но одноимённых величин. Надо, однако, помнить, что термин «децибел» всегда связывают только с мощностями и с некоторыми оговорками с напряжениями и токами. Физическая природа мощностей не оговаривается и может быть любой - электрической, акустической, электромагнитной.

    Децибел, как показывает приставка «деци», представляет собой десятую часть другой, более крупной единицы - Бел. А Бел есть десятичный логарифм отношения двух мощностей. Если известны две мощности Р1 и Р2, то их отношение, выраженное в децибелах, определяется как:

    N дБ =10 Lg (P2/P1)

    где Р1 - мощность, соответствующая начальному уровню сигнала, а Р2 - мощность, соответствующая конечному уровню сигнала.

    Здесь уместно напомнить, что десятичным логарифмом числа называют показатель степени, в которую надо возвести число 10, чтобы получить данное число. Например: Lg(100) = 2, так как 10 2 = 10*10 = 100; Lg(1000) = 3, так как 10 3 = 10*10*10 = 1000.

    У чисел, которые больше единицы, логарифмы будут положительными величинами, а если числа меньше единицы, их логарифмы отрицательны. Перед отрицательными логарифмами ставят знак «-» (минус), например: Lg(0,1) = - 1; Lg(0,01) = - 2.

    В том случае, когда начальный сигнал меньше конечного, то есть P2/P1 больше 1, что имеет место в усилителях, число децибел будет положительным, а если начальный уровень больше конечного, то есть P2/P1 меньше 1, то число децибел будет отрицательным. Второй случай соответствует ослаблению (затуханию) сигнала. Когда обе мощности одинаковы и P2/P1= 1, то число децибел равно нулю.

    Между децибелами усиления и затухания существует простая связь: если, к примеру, отношение 10 соответствует 10 дБ, то -10 дБ выражают обратное отношение, то есть 0,1.

    Сравнение двух сигналов путём сопоставления их мощностей не всегда бывает удобным. Во многих случаях оказывается проще измерять не мощность в нагрузке, а падение напряжения на ней или протекающий ток. Но при этом нужно соблюдать обязательное условие: сопротивления нагрузок, на которых измеряются напряжения U1 и U2 или через которые протекают измеряемые токи I1 и I2 должны быть одинаковыми. Формулы для расчёта децибел в этом случае имеют следующий вид:

    N дБ =20 Lg (P2/P1); N дБ =20 Lg (I2/I1)

    Децибелы применяют не только для сравнения двух величин. Они удобны и для оценок конкретных значений мощностей, а также напряжений и токов, если считать, что величина одного из членов отношения, входящего в приведённые выше формулы, неизменна. Тогда любая другая величина, сравниваемая с нею, будет характеризоваться определённым числом децибел. В этом случае нулю децибел соответствует мощность, равная первой, которую часто называют нулевой. За условный нулевой уровень электрического сигнала принята мощность Р = 1 мВт (0,001 Вт), выделяемая на активном сопротивлении R = 600 Ом - подобно тому, как при измерении температуры за нуль градусов принята температура таяния льда при нормальном атмосферном давлении. При этой мощности на указанном сопротивлении падение напряжения равно:

    U = (РR) 0,5 = (0,001*600) 0,5 = 0,775 В,

    а протекающий ток:

    I = (P/R) 0,5 = (0,001/600) 0,5 = 1,29 мА.

    Эти величины - 0,775 В и 1,29 мА приняты за нуль децибел электрического напряжения и тока.

    Если в цепи с активным сопротивлением 600 Ом выделяется мощность больше 1 мВт, то есть падение напряжения больше 0,775 В и ток больше 1,29 мА - уровни будут положительными. Когда же мощность, напряжение или ток меньше этих величин, то уровни отрицательные.

    Децибелы и соответствующие им отношения мощностей, напряжений и токов даны в табл. 1.

    Допустим, что в результате усовершенствования оконечного каскада усилителя низкой частоты его выходная мощность возросла с 10 до 20 Вт. Значит приращение мощности будет:

    P2/P1 = 20/10 = 2

    По таблице в колонке «Отношение мощностей» ближайшее к 2 число будет 1,99. В колонке «Децибелы» этому числу соответствует 3 дБ. Следовательно, увеличение выходной мощности вдвое соответствует увеличению усиления на 3 дБ. Если же по каким-либо причинам выходная мощность усилителя снизилась с 20 Вт до 10 Вт, то новое отношение мощностей будет P2/P1 = 10/20 = 0,5. Но теперь изменение мощности означает ослабление и будет выражаться как -3 дБ.

    Выполняя действия с децибелами, надо помнить, что сумма двух чисел в децибелах эквивалентна произведению абсолютных величин тех чисел, которым они соответствуют, поэтому, чтобы показать рост (или ослабление) мощности, например, вдвое, втрое или вчетверо, надо к первоначальному числу децибел прибавить (или отнять) соответственно 3 дБ, 4,8 дБ или 6 дБ.

    В децибелах часто выражают чувствительность микрофонов,сравнивая отдаваемую ими мощность при заводских испытаниях с указанным выше стандартным нулевым уровнем 1 мВт. Допустим, что микрофон типа МД-44, выходной уровень отдачи которого - 78 дБ, подключён к усилителю, который может развивать 40 Вт неискажённой мощности. Однако в работе оказалось, что усилитель с таким микрофоном развивает только 10 Вт. Спрашивается, какой чувствительности должен быть микрофон, чтобы усилитель отдавал полную мощность? Отношение максимальной мощности (40 Вт) усилителя к получаемой (10 Вт) составляет 40/10 = 4. Этому отношению (по таблице - 3,98) соответствует 6 дБ. Следовательно, нужен микрофон с уровнем отдачи - 72 дБ, то есть на 6 дБ больше, чем микрофон МД-44 (-78 дБ), так как: - 78 дБ + 6 дБ= -72 дБ. Этому требованию отвечает, например, микрофон МД-41.

    Таблица 1. Децибелы и соответствующие им отношения мощностей, напряжении и токов

    Децибелы Отношение мощностей Децибелы Отношение мощностей Отношение напряжений или токов
    -60 0,000001 0,001 6,0 3,98 1,99
    -50 0,00001 0,003 6,2 4,17 2,04
    -40 0,0001 0,01 6,4 4,36 2,09
    -30 0,001 0,032 6,6 4,57 2,14
    -20 0,01 0,10 6,8 4,79 2,19
    -10 0,10 0,30 7,0 5,01 2,24
    -6 0,25 0,50 7,2 5,25 2,29
    -3 0,50 0,70 7,4 5,50 2,34
    -2 0,63 0,80 7,6 5,75 2,40
    - 1 0,80 0,90 7,8 6,03 2,46
    0 1,00 1,00 8,0 6,31 2,51
    1,0 1,26 1,12 8,2 6,61 2,57
    1,2 1,32 1,15 8,4 6,92 2,63
    1,4 1,38 1,17 8,6 7,24 2,69
    1.6 1,44 1,20 8,8 7,59 2,75
    1.8 1,51 1,23 9,0 7,94 2,81
    2,0 1,58 1,26 9,2 8,32 2,88
    2,2 1,66 1,29 9,4 8,71 2,95
    2,4 1,74 1,32 9,6 9,12 3,02
    2,6 1,82 1,35 9,8 9,55 3,09
    2,8 1,91 1,38 10,0 10,00 3,16
    3,0 1,99 1,41 11,0 12,59 3,55
    3,2 2,09 1,44 12,0 15,85 3,98
    3,4 2,19 1,48 13,0 19,95 4,47
    3,6 2,29 1,51 14,0 25,12 5,01
    3,8 2,40 1,55 15,0 31,62 5,62
    4,0 2,51 1,58 16,0 39,81 6,31
    4,2 2,63 1,62 17,0 50,13 7,08
    4,4 2,75 1,66 18,0 63,10 7,94
    4,6 2,88 1,70 19,0 79,43 8,91
    4,8 3,02 1,74 20,0 100,00 10,00
    5,0 3.16 1,78 30 0 1000,00 31,62
    5,2 3,31 1,82 40,0 10000,00 100,00
    5,4 3,47 1,86 50,0 100000,00 316,00
    5,6 3,63 1,91 60,0 1000000,00 1000,00
    5,8 3,80 1,95

    Ещё пример. К отрезку кабеля типа РК-1 длиной 50 м приложено напряжение 8 В частотой 100 МГц. Каким будет напряжение на выходе отрезка, если известно (из справочника), что на этой частоте кабель вносит затухание 0,096 дБ на метр? Источник питания и нагрузка имеют одинаковые сопротивления, равные волновому. Очевидно, что затухание, вносимое кабелем, равно: 0,096*50 = 4,8 дБ. В табл. 1 для этого затухания (-4,8 дБ) величина отношения напряжений не указана. Воспользуемся тем, что в таблице приведено отношение для +4,8 дБ, которое равно 1,74. Значит на конце отрезка сигнал будет составлять 1/1,74 ≈ 0,57 от входного, т. е. 8*0,57 ≈ 4,6 В.

    Когда надо определить значения децибел или отношений, которых нет в таблице, надо поступать следующим образом. Предположим, необходимо найти отношение мощностей, соответствующее 24 дБ. Представив 24 дБ в виде суммы 10 + 14 дБ, найдём в таблице отношения мощностей для каждого из слагаемых, они равны 10 и 25,12. Перемножив эти отношения, получим, что 24 дБ соответствуют отношению мощностей 251,2.

    На выходе усилителя на средних частотах развивается напряжение U1 = 30 В, а на краях полосы пропускания - напряжение U2 = 21 В. Усилитель, следовательно, вносит частотные искажения - верхние и нижние звуковые частоты усиливает хуже («заваливает»), чем средние. Отношение указанных величин будет

    U2/U1 = 21/30 = 0,7

    По таблице найдём, что частотные искажения данного усилителя на краях полосы пропускания равны -3 дБ.

    Широко применяются децибелы и в акустике, где они являются, по существу, основной единицей для количественной оценки интенсивности звука. Объясняется это свойством нашего уха реагировать на звуки, интенсивность которых отличается в миллионы раз. Но чувствительность уха к звукам разной силы не одинакова - в тишине и при малой интенсивности (шёпот, шорохи) она максимальна, а при большой интенсивности (рёв самолёта, грохот машин) она минимальна. В этом отношении слуховой аппарат подобен радиоприёмнику с системой АРУ.

    Это явление можно пояснить таким примером. Допустим, усилитель развивает на выходе мощность 10 Вт. Увеличение выходной мощности до 20 Вт покажется на слух небольшим увеличением громкости. Для того чтобы ухо ощутило вдвое большую громкость, понадобится почти десятикратное увеличение выходной мощности усилителя (≈10 дБ). А чтобы ухо восприняло увеличение громкости в 4 раза, мощность должна быть увеличена в 100 раз (≈20 дБ).

    Учёные-физиологи, исследуя свойства слуха, установили, что чувствительность уха связана с интенсивностью звукового воздействия логарифмическим законом, то есть приращение силы звука в несколько раз покажется на слух изменением громкости приблизительно в логарифм этого числа раз. Применение децибел в акустике оказывается очень удобным, так как слуховое восприятие и оценка интенсивностей звуков при этом находятся в строгой связи и, к тому же, изменение интенсивности звука на 1 дБ улавливается ухом как едва заметное изменение громкости.

    Таблица 2. СРЕДНИЕ УРОВНИ ШУМОВ

    Субъективная оценка шума Уровень шума (дБ) Источники или место измерения шума
    оглушительные - 130 - Болевом порог (звук воспринимается как боль)
    Гром над головой
    Пушечный выстрел
    Клепальная машина
    Очень шумный цех
    - 120 -
    - 110 -
    Очень громкий - 100 - Симфонический оркестр (пики громкости)
    Деревообрабатывающий цех
    Уличный громкоговоритель
    Шумная улица
    Металлообрабатывающий цех
    - 90 -
    Громкий - 80 - Свисток милиционера (15м)
    Радиоприёмник громко (2,5м)
    Машинописное бюро
    Спокойный разговор (4м)
    Зал большого магазина
    - 70 -
    Умеренный - 60 - Тихая улица большого города
    Учреждение средней шумности
    Ресторан
    Легковая машина (10-20м)
    Жилое помещение
    - 50 -
    Слабый - 40 - Читальный Зал
    Тихий разговор
    Шелест бумаги
    Шёпот
    Больничная палата
    - 30 -
    Очень слабый - 20 - Тихая ночь за городом
    Заглушённая комната
    Порог слышимости
    - 10 -
    - 0 -

    Сравнительная оценка средних уровней громкости некоторых бытовых и производственных шумов в децибелах относительно порога слышимости человеческого уха, принятого за нулевой уровень, приведена в табл. 2. Измерение интенсивности звука осуществляется с помощью специальных приборов - шумомеров, шкалы которых градуированы непосредственно в децибелах.

    Приведёнными здесь примерами далеко не исчерпываются применения децибел при различных подсчётах и измерениях в радиолюбительской практике. Мы лишь хотели показать простоту понимания децибела и широкие возможности пользования ими.

    Канд. техн. наук Е. ЗЕЛЬДИН, инж. К. ДОМБРОВСКИЙ

    Электротехники пользуются различными параметрами при знакомстве с электрической схемой, измерении абсолютных величин напряжения, сопротивления, индуктивности. Каждое условно обозначено, их легко определить от начального уровня отсчета до измеренной величины, например, силы тока в амперах. Но встречаются и надписи рядом с емкостью, частотой или другим электрическим параметром, которые имеют количественное обозначение, в виде децибел.

    Использование базовой единицы дБ широко применяется проектировщиками телекоммуникаций для сравнения характеристик в различном оборудовании. С помощью специфической единицы численно выраженной определяют, насколько усилен или ослаблен сигнал.

    Часто измеряя какой-то электрический размер, в представлении встают прямые единицы, но специалисты оценили использование относительной шкалы, в которой децибелы встают в качестве мощного инструмента, пришедшего в науку благодаря ученому Грэхему Беллу.

    Его мировая известность стала после изобретения телефонов, но он много провел исследовательских работ, определяя слуховой порог у человека. Для этого ученым была основана Ассоциация для людей, действующая поныне.

    Кто создал шкалу для измерения громкости


    Александр Белл стал первым в количественном определении слухового чувства. Он установил зависимость слуховой восприимчивости от логарифмов, вместо мощностей звуковых волн. Исследователь обнаружил пороги слышимости детей и взрослых, определил, при каких значениях происходит, что нормальному человеку достаточно 13 граней диапазона, чтобы громкость воспринималась без ущерба для здоровья.

    Эти данные стали основой для производства шкалы логарифмов, в ней звуковая мощность разделена на единицы, которые назвали белами. По такому разграничению установлено, что тихий шепот равен трем белам, а нормальная речь шести. Подобное логарифмическое измерение является его стандартной величиной, определяемой отношением энергий в двух уровнях. В расшифровку, что измеряется в децибелах вложено общее выражение громкости, возведен бел в десятые доли для удобства обращения с цифрами.

    Понятия о звуке, уровне шумов и их источниках

    По физическим характеристикам звук и шум отличаются своими особыми природными явлениями. Изменяется давление в воздухе, который действует на барабанные перепонки в ушах с помощью своеобразных колебаний, происходит принятие звука. Он продолжает движение по органам человека преобразованными электрическими импульсами, достигая. Человек способен принимать широкий диапазон звукового давления, который выражен децибелами.

    Распространение звуков происходит по различным частотам, что отражается на чувствительности ушей как животных, так и людей.
    Отличия между звуками и шумами чисто субъективные, которые определены источниками возникновения. В зависимости от того, какая среда окружает человека, они бывают внутренними, связанными с оборудованием:


    • инженерным
    • технологическим
    • бытовым
    • санитарно-техническим

    К внешним источникам относятся шумы, возникшие от:

    • транспортных средств
    • промышленных организаций
    • энергетических предприятий
    • различных учреждений зависящих от жизнедеятельности людей (стадионы, спортивные площадки, развлекательные мероприятия)

    Слышен шум в квартирах, порой достигающий 60 дБ, от санитарного и инженерно-технического оборудования:

    • лифтов
    • насосов
    • мусоропроводов
    • вентиляционных установок

    В домах слышат:

    • музыкальную аппаратуру
    • в рабочем режиме приборы и инструменты
    • бытовую технику

    При передвижениях по квартире (двигая громоздкие предметы) происходит возникновение звуковых колебаний, переходящих в структурный шум. Работа вентиляторов, лифтовых лебедок в зданиях источают как структурный, так и воздушный шумовой поток, который через вентиляционные каналы попадает в помещения.

    Чтобы не было слышно работы механического оборудования, устанавливают виброизолирующие приспособления. В многоэтажках от движения между этажами лифтовой лебедки, ударных и толчковых действий направляющих, бряцанья дверных створок, распространение звуковых эффектов происходит как воздушным путем, так и конструктивным.

    Помимо раздражающего воздействия на организм шумов в домашних условиях, человек подвергается превышению допустимых норм на государственной службе , во время производственных процессов.


    Промышленность обладает изобилием создаваемых в ходе производства шумов. Часто от сжатого воздуха слышен рокот производства. Он называется импульсным: возникновение происходит, когда продувают клапаны и цилиндры, чистят оборудование, охлаждают его, транспортируют, сортируют.

    Если для звуков характерны определенные тембры, спектральные окраски и люди легко узнают источники их. К примеру, звучание музыки, детский крик, лай собаки. То шумы, поступающие от случайных колебательных и непериодических процессов, не имеют определенных источников. Это может быть гомон толпы, треск строительных площадок, гудение машин, галдеж улицы.

    Поэтому, определяя шум как явление, его сравнивают с комплексом бесконтрольных звучаний, неблагоприятно воздействующих на здоровье человека, раздражающих, мешающих приятному времяпровождению. Их классифицируют на типы:

    • воздушные
    • конструкционные
    • ударные

    По воздуху распространяются помехи от телевидения, радио, ссоры соседей. Структурным способом передается треск перегородок, скрип половых, потолочных конструкций в домах, слышимость от работающих механизмов, шуруповерта, процессора, пылесоса. К разновидностям, конструкционных беспорядочных звуковых колебаний, относятся ударные звуки. Их слышно из соседних квартир расположенных на верхних этажах, когда падает стул, передвигают мебель.

    Действия шумов, допустимая их величина


    Вид любого шумового загрязнения проявляется как увеличивающийся звуковой уровень , выше существующего в природе, вызывающий раздражительные факторы. Звуковые сигналы в совокупности дают живому существу время оценить их, адаптироваться.

    Большая мощность вызывает поражение слуховых органов, происходит ощущение боли, шокирующие действия. Инновационные разработки привели к угрожающим масштабам увеличения шумной среды, допускающей не только раздражающие эмоции, но и снижения слуховой остроты. От нарушения акустического комфорта у человека возникают стрессы, бессонницы, повышение давления.

    Главная угроза состоит в частичной или полной потере слуха.

    Грохот, визг, лязг в повышенных тонах ведет к рассеиванию внимания, снижению трудоспособности, результативности от трудовой деятельности, в особенности, если это умственный труд. Человек теряет возможность сосредотачиваться на главной операции, принимать важные решения. Нарушается нормальная, источаются нервные клетки. От их ослабления происходит сбой в координации различных органов.

    Примеры на человеческий организм шумовых воздействий

    Средой для слуха является уровень громкости до 30 дБ. Существует допустимая граница, не превышающая 80 дБ, но после 60 дБ человек начинает некомфортно себя ощущать.

    Увеличение звука до 120 дБ вызывает боль, после 140 дБ происходит непереносимое чувство. Даже металл не выдерживает 180 дБ, возникает его усталость, а увеличивая уровень, может произойти разрушение конструкции. Шумные производства славятся своей громкостью до 110 дБ. Квартиры всегда волнуют многих жителей своей звукоизоляцией, над этим работают целые конструкторские бюро, разрабатывая методы и новые шумопоглощающие материалы.

    Известно, что повышение шумов:


    • от 61 дБ - расстраивает вегетативную нервную систему
    • 91 дБ - снижает слух
    • 116 дБ - считается болевым порогом
    • 140 дБ - вызывает разрыв барабанных перепонок
    • 151 дБ - является нестерпимым
    • 179 дБ – угроза жизни

    Неблагоприятно действует на умственное развитие детей превышение допустимых шумовых норм . Вредно для подросткового возраста часто посещать дискотеки, где музыка звучит до 100 дБ, иногда специально ставят усилители и грохот становится равным электропоезду.

    Шумовой фон мегаполисов увеличивается пропорционально с постоянно повышающимися технологическими процессами. Появилось много, заставляющих решать задачи исследователям, разрабатывать различные нормативные акты , чтобы обезопасить человека от звуковых воздействий.

    С помощью звукоизоляционной продукции, отражающей энергию звука, защищают здания. Все они гибкие, упругие и многослойные, выполняют основную задачу, закупоривают поверхности, не пропускают звуки.

    Как действуют на слуховой орган раздражающий грохот, умиротворяющие звуки


    Невозможно жизненное пространство людей полностью оградить от звукового фона . Среди них имеются полезные сигналы, которые благоприятно действуют на человека. С их помощью общаются, ориентируются, трудятся.

    Известно, что благотворно для нервной системы журчат ручьи, поют птицы, шелестят листья. Нервный стресс могут снять нежная песня, рокот морских волн. Звуками наказывали в средневековье, обрекая приговоренного находиться долгое время под ударами колокольного звона. Гармоничной колыбельной заставляли успокоиться и заснуть даже беспокойного ребенка.

    Поступление некоторых звуковых сигналов в мозг человека вызывают неприятные ощущения, раздражения, утомления. Люди испытывают субъективные ощущения от услышанного, а также в органах слуха могут произойти патологические изменения.

    Раздражающий гомон может подействовать на системы:

    • центральную
    • нервную
    • сердечно-сосудистую
    • эндокринную
    • пищеварительную

    Определено, что на человека повышенный шумовой уровень действует следующим образом:

    • понижается слуховая функция, адаптация от слухового утомления вплоть до частичной или постоянной потери слуха
    • нарушается способность общаться с помощью речи
    • раздражительное, беспокойное поведение
    • изменяются физиологические реакции,
    • ухудшается психика
    • уменьшается производительность

    Как специфические раздражители слуховых органов, с определенной частотой и интенсивностью служат действия звуковых волн.

    Исследования ученых на действие шума

    Влияние на слух громких сигналов побудили человека изучать их характеристики теоретически и практически. Цель такого исследования выявить порог угрожающего действия шума, на этом основании разработать документы и обосновать гигиенические нормы различного контингента жителей, в зависимости от того, где они находятся и в каких пребывают.


    Это может быть:

    • жилой дом
    • общественное здание
    • производственное сооружение
    • образовательное учреждение
    • больница, поликлиника
    • профилактические заведения
    • спальный район
    • промышленный округ
    • территория отдыха

    В теории ученые справились с задачей по изучению патогенеза, способам воздействия шумов, адаптации организма в неблагоприятной среде, последствиям от длительного пребывания в ней. Проводились многочисленные эксперименты. Это сложная исследовательская работа, так как имеются значительные отличия к шумовой чувствительности граждан от их возраста, пола, социальной группы.

    Человек по-разному реагирует на звуковые эффекты в зависимости от того, в каком состоянии он находится - возбужденном или заторможенном; какой процесс преобладает в этот момент.

    В процентном отношении люди подразделяются на восприятие звуков с чувствительностью:

    • 35% - повышенной
    • 55% - нормальной
    • 10% - не воспринимают шум

    Акустический стресс влияет на психологическое и физиологическое состояние жителей и зависит от области:

    • индивидуального биоритмического профиля
    • характера сна
    • физической активности
    • стрессов
    • нервного состояния
    • употребления алкоголя и курения

    Социологи утверждают, что больше всего городские граждане сетуют на гудение машин (70%), грохот от промышленных предприятий занимает среднюю строчку (20%), домашний гомон стоит на последнем месте (10%). При этом 50% ощущают беспокойство, 30% раздражаются, а 20% вообще не жалуются. Страдают граждане, у кого поражена нервно-сосудистая система или органы пищеварения.

    От этого происходят или обостряются заболевания:

    • гастрита
    • кишечника

    У жителей, проживающих постоянно в районе улиц с повышенным уровнем шума, ухудшается состояние здоровья, увеличивается количество обращений к врачебной помощи.

    Основные правила, способные уберечь слух

    Человеку, имеющему бесценный дар слышать, трудно представить, что можно потерять его. Для этого имеются профилактические меры, предупреждающие неприятные последствия:


    1. Лечение. Особенно актуально в детском возрасте. При ушных инфекциях бактериального характера необходимо компетентное и своевременное лечение. Многие заболевания несут кроме инфекций опасные осложнения слухового аппарата.
    2. Уменьшить посещение заведений, где повышен уровень шума. В ресторанах, барах, концертных залах люди переговариваются на повышенных тонах, стоит подумать о выборе места и длительности пребывания там.
    3. Не нужно пренебрегать защитными средствами людям, у которых деятельность проходит в повышенном шумовом уровне , а свыше 80 дБ это очень громко.
    4. Вредно долгое использование наушников.
    5. Громкость от радио, телевизоров, магнитол стоит приглушать по возможности.
    6. Чистота залог здоровья, а уши необходимо постоянно очищать от серных скоплений, использование ватных палочек не одобряется врачами, они рекомендуют промывать водой.

    Просты, а сухие технические знания о децибелах помогут решить практическую задачу, но не сохранят человеку слух. Необходимо бережно относиться к своему здоровью, избегать шумных сборищ, защитить свое жилище от проникновения посторонних звуков. Специалисты всегда придут на помощь: врач вылечит, строитель произведет монтаж звукоизоляции.

    Ноя 28, 2016 Виолетта Лекарь

    Децибел - это десятая часть Бела, логарифмической единицы, названной в честь изобретателя телефона Александра Грэхема Белла (1847-1922). Один Бел соответствует десятикратному увеличению мощности сигнала: 10 дБ = 1 Б = Ig10. Десятикратному ослаблению мощности соответствует -10 дБ = -1 Б = Ig0,1. Однако напряжение или ток при десятикратном изменении мощности изменяются только в 3,16 раза (мощность пропорциональна квадрату напряжения или тока). Таким образом, усиление G или ослабление а, выраженное в децибелах, равно:

    G, α(дБ) = 10lg(P2/P1) = 20lg(U2/U1).

    Предостережем от распространенных ошибок: не бывает "децибелов по напряжению" и "децибелов по мощности" - усилитель, имеющий G = 20 дБ, усиливает мощность сигнала в 100 раз, а напряжение (при равенстве входного и выходного сопротивлений) - в 10 раз. Оговорка в скобках существенна - ведь переменные напряжения и токи можно трансформировать, оставляя при этом неизменной мощность. Никому не придет в голову сказать, что трансформатор, повышающий напряжение в 10 раз, имеет усиление 20 дБ. Его усиление G = 0 дБ, или даже α = - 0,1...1 дБ, если учесть незначительные потери. Итак, чтобы пользоваться формулой

    G = 20lg(U2/U1),

    надо сначала привести входное U1 и выходное U2 напряжения к одинаковым сопротивлениям, формулой же G или α = 10lg(P2/P1) пользуются без ограничений.

    Оказалось, что в децибелах чрезвычайно удобно измерять громкость звука, мощность и напряжение сигнала, усиление и ослабление (затухание) любых цепей, длинных линий и фильтров. Первыми стали широко пользоваться децибелами именно телеграфисты и телефонисты - для оценки затуханий и уровней сигнала в линиях. Главное достоинство оказалось в том, что при расчетах умножение и деление заменяется сложением и вычитанием, которые легко сделать даже в уме, а на графиках, построенных в логарифмическом масштабе, многие кривые становятся прямыми.

    Для отсчета любой величины в децибелах нужен исходный (нулевой) уровень. При расчете усиления и ослабления исходным уровнем служит значение рассматриваемой величины на входе устройства (Р1, U1). Если же мы имеем дело с определенными, конкретными величинами, имеющими размерность (логарифм можно взять только от безразмерного числа), то исходный уровень надо задать.

    Нулевой уровень громкости соответствует усредненной пороговой чувствительности человеческого слуха, при которой сила звука (плотность потока акустической энергии) составляет 10-12 Вт/м2, а звуковое давление - 2·10-5 Па. Это чрезвычайно малые величины. Так, например, скорость колеблющихся частиц воздуха при такой силе звука составляет всего 5·10-8 м/с, а смещение этих частиц от положения равновесия (при частоте звука 1000 Гц) - всего 2·10-11 м, что сравнимо с размерами молекул! Вот какой совершенный орган слуха создала природа.

    Допустим, ваш громкоговоритель развивает стандартное звуковое давление 0,2 Па (на расстоянии 1 м при подводимой электрической мощности 0,1 Вт), что соответствует силе звука (определяется по справочнику) 10"4 Вт/м2. Найдем громкость в децибелах:

    10lg(10-4/10-12) = 80 дБ, что примерно соответствует громкости звучания оркестра. Можно обойтись и без справочника, используя данные по звуковому давлению, учтя, что сила звука и громкость пропорциональны квадрату звукового давления (так же, как мощность пропорциональна квадрату напряжения): громкость = 20lg(0,2/2·10-5) = 80 дБ. Для ориентировки приведена табл. 1, связывающая громкость, силу звука и звуковое давление.


    Надо заметить, что шкала громкости в децибелах имеет мощное физическое, даже лучше сказать, физиологическое обоснование. Дело в том, что характеристика субъективного восприятия громкости нелинейна - она подчиняется логарифмическому закону (так же, впрочем, как и характеристики других органов чувств). Это значит: для того, чтобы вызвать заметное увеличение громкости при малых уровнях, надо добавить совсем немного мощности, а при больших уровнях - довольно много. Однако в процентах к исходному уровню прибавка составит одну и ту же величину, например, 26 %. В децибелах зто будет 10lg(1.26/1) = 1 дБ. В этом и заключается "секрет" логарифмических шкал - увеличение аргумента на сколько-то вызывает изменение функции во сколько-то раз.

    Силу звука в табл. 1 тоже можно выразить в децибелах, и для частоты 1000 Гц значения будут совпадать со значениями громкости. На других частотах звукового диапазона чувствительность человеческого слуха несколько иная, и при равной силе звука субъективно воспринимаемая громкость, как правило, меньше. Зависимость между силой звука и громкостью для различных частот (цифры около кривых) представлена на рис. 36.

    Обратная логарифмической, экспоненциальная зависимость встречается в природе гораздо чаще, чем линейная. Давление воздуха в атмосфере понижается в е раз (е = 2,72 - основание натуральных логарифмов) при подъеме на каждые следующие 8 км, число радиоактивных атомов и их масса уменьшаются вдвое по прошествии времени, равного периоду полураспада, и т. д. Все подобные зависимости на графиках, построенных в логарифмическом масштабе, отображаются прямыми линиями.

    Мощность часто измеряют относительно уровня 1 мВт. Этот "нуль" принят как стандартный телефонный уровень, соответствующий напряжению 0,775 В на нагрузке 600 Ом. Им чрезвычайно часто пользуются и в технике сверхвысоких частот (СВЧ). Чтобы указать на этот нулевой уровень, используют (вместо дБ) обозначение дБм:

    Р(дБм) = 101д(Р/1мВт).

    Мощность в 1 мВт соответствует 0 дБм, 1 Вт - +30 дБм, 0,1 мВт - -10дБм. Точно так же напряженность поля часто отсчитывают от уровня 1 мкВ/м, например, напряженность поля 46 дБмкВ соответствует 200 мкВ/м.

    Для облегчения перевода величин в децибелы и обратно полезна табл. 2. В ней даны только единицы децибел, с десятками дело обстоит гораздо проще. Каждые 10 дБ дают увеличение мощности в 10 раз и напряжения - в 3,16 раз. Пусть требуется узнать, во сколько раз уменьшаются мощность и напряжение сигнала на выходе фильтра с затуханием 48 дБ. Заметим, что 48 = 40 + 8, 40 дБ дают ослабление в 10000 раз, а 8 дБ - еще в 6,3 раза. Следовательно, мощность на выходе фильтра уменьшается в 63 000 раз. Уменьшение напряжения можно узнать, если извлечь квадратный корень из этого числа. Получится 250 - ведь мощность пропорциональна квадрату напряжения. Но мы продолжим расчет в децибелах. 40 дБ дают 100 раз и 8 дБ - 2,5 раза. Опять получается 250 раз.

    Другой пример. Усилитель имеет коэффициент усиления 17 дБ, входное и выходное сопротивления равны, во сколько раз усиливается напряжение? В таблице нет 17 дБ, но 17 = 20 - 3.

    Усиление в 20 дБ соответствует увеличению напряжения в 10 раз, а - 3 дБ означает ослабление в 1,4 раза. Итого: 10/1,4=7. Найдем ответ иначе: 17 = 8 + 9; 8 дБ соответствуют увеличению напряжения в 2,5 раза, а 9 дБ - в 2,8. Перемножим в уме эти числа и получим 2,5·2,8 = 7.

    В заключение приведем полезный график, относящийся к материалу, изложенному в разделе "Этот непростой закон Ома " ("Радио", 2002, № 9, с. 52, 53). Там мы рассматривали простейшую цепь, состоящую из генератора с внутренним сопротивлением r и нагрузки сопротивлением R. Было показано, что максимальная мощность отдается в нагрузку при равенстве сопротивлений r = R. А что будет при их неравенстве? Отдаваемая в нагрузку мощность окажется меньше, но насколько? На рис. 37 дан ответ в децибелах в зависимости от коэффициента рассогласования k = r/R.


    Вопрос для самопроверки . Получите формулу зависимости отдаваемой в нагрузку мощности в зависимости от коэффициента рассогласования к, и затем постройте график, аналогичный рис. 37. Подумайте, какие сведения на этом графике являются избыточными и что надо сделать, чтобы упростить его?

    Ответ . Для простой цепи, содержащей источник с ЭДС Е и внутренним сопротивлением г и нагрузку сопротивлением R (рис. 4), ток равен l = E/(r + R).

    Это справедливо и для постоянного, и для переменного тока. Напряжение на нагрузке составит U = ER/(r+R).

    Найдем мощность в нагрузке P = U·l = E 2 R/(r+R) 2 .

    При равенстве сопротивлений нагрузки и источника (R = r) эта мощность максимальна и составляет Р 0 = Е 2 /4r.

    Найдем потери при рассогласовании P/P 0 = 4rR/(r + R) 2 .

    Если разделить и числитель и знаменатель правой части формулы на R 2 и учесть, что r/R = k (коэффициент рассогласования), то получим P/P 0 = 4k/(1+k) 2 .

    Это и есть та формула, по которой построен график рис. 37. Разумеется, формула дает отношение Р/Р 0 "в разах", а на графике оно уже переведено в децибелы. Поясним примером: при k = 2 отношение мощностей составит Р/Р 0 = 8/9. С помощью логарифмической линейки (которой автор до сих пор с большим успехом пользуется несмотря на наличие нескольких калькуляторов и компьютера) в доли секунды находим потери из-за рассогласования - 0,5 дБ.

    Любопытно отметить, что подстановка k = 0,5 дает абсолютно то же самое значение потерь. Значит, рассогласование нагрузки вдвое (как в сторону ее уменьшения, так и увеличения) дает одинаковое уменьшение мощности в нагрузке. Это действительно так, и выведенная нами формула останется той же самой при подстановке k"= 1/k. Имейте в виду, что в литературе часто встречается и другое определение коэффициента рассогласования: k"= R/r, но результаты расчета потерь оказываются одинаковыми.

    Таким образом, график на рис. 37, построенный в логарифмическом масштабе, симметричен относительно точки к = 1. Вполне можно было обойтись одной его половиной, взяв значения к либо меньше, либо больше единицы и указав на оси абсцисс "к или 1/к". В этом и состоит избыточность графика.

    Как видим, даже при довольно значительном рассогласовании (сопротивление нагрузки отличается от внутреннего сопротивления источника в два раза) потери из-за рассогласования весьма невелики. Если, например, мы имеем дело с усилителем звуковой частоты , то изменение громкости на 0,5 дБ практически не уловимо на слух. В области больших рассогласований (к " 1 или к " 1) потери мощности из-за рассогласования уже значительны.